Online support at PGPYREXIA AIPGMEEE EXPLORER...

103
PGPYREXIA AIPGMEEE EXPLORER 2011 Online support at www.pgpyrexia.co.cc PGPYREXIA REVIEW OF ALL INDIA POSTGRADUATE ENTRANCE EXAMINATION 2011 AUTHORED BY : PGPYREXIA TEAM

Transcript of Online support at PGPYREXIA AIPGMEEE EXPLORER...

PG

PY

RE

XIA

AIP

GM

EE

E E

XP

LOR

ER

20

11

On

lin

e s

up

po

rt

at

ww

w.p

gp

yr

ex

ia.c

o.c

c

PGPYREXIA REVIEW OF ALL INDIA

POSTGRADUATE ENTRANCE

EXAMINATION 2011

AUTHORED BY :

PGPYREXIA TEAM

Online support at www.pgpyrexia.co.cc

Free monthly AIIMS/ALL INDIA mock exams @ www.pgpyrexia.lefora.com.

Visit today!! Page 2

CONTENTS:

SUBJECT PAGE NUMBER

ANATOMY 3

PHYSIOLOGY 12

FORENSIC MEDICINE 16

MICROBIOLOGY 19

PHARMACOLOGY 29

SOCIAL AND PREVENTIVE MEDICINE 44

ORTHOPAEDICS 52

OPHTHALMOLGY 56

ENT 60

PAEDIATRICS 63

BIOCHEMISTRY 67

SURGERY 74

PATHOLOGY 87

OBSTRETICS AND GYNAECOLOGY 102

DERMATOLOGY 112

ANAESTHESIOLOGY 114

PSYCHIATRY&RADIOLOGY 118

MEDICINE AND MISCALLENOUS 123

Get free MCQs, High Yield Facts, Exam Notifications, Book Alerts and other PG news &

updates in your mobile. SMS ON pgpyrexia to 09870807070

Online support at www.pgpyrexia.co.cc

Free monthly AIIMS/ALL INDIA mock exams @ www.pgpyrexia.lefora.com.

Visit today!! Page 3

A kind request to readers:

We are very happy to provide you with resource that helps you in your battle for a PG seat.If you want to appreciate our work and help us do

better, kindly recall the Questions of upcoming AIIMS and PGI exams by contributing in our “EXAM RECALLER” section of pgpyrexia.co.cc

Looking forward to your participation!!

ANATOMY:

1. All are TRUE regarding injury to common peroneal nerve EXCEPT?

A. Loss of sensation over sole

B. Foot drop

C. Injury to neck of fibula

D. Loss of dorsiflexion of toe

Ans: A. Loss of sensation over sole

Ref: Snell’s anatomy

“The common peroneal nerve is extremely vulnerable to injury as it winds around the neck of the fibula. At this site, it is exposed to direct trauma or is involved in fractures of the upper part of the fibula. Injury to the common peroneal nerve causes footdrop. the patient will be unable to dorsiflex the ankle joint (which showed footdrop) and evert the foot. In addition, there will be evidence of diminished sensation down the anterior and lateral sides of the leg and dorsum of the foot and toes, including the medial side of the big toe.

2. In L5 root involvement, which among the following is not affected?

A. Thigh adduction

B. Knee flexion

C. Knee extension

D. Toe extension

Ans: A. Thigh adduction

L5 nerve involvement causes foot drop and in a patient with foot drop, weakness of thigh adduction localizes the lesion to lumbosacral plexus

(Since thigh adduction is due to L2 and L3 and not due to L5)

3. All are seen in the floor of 3rd ventricle except?

A. Infundibulum

B. Oculomotor nerve

C. Mammillary body

D. Optic chiasma

Ans: B. Oculomotor nerve

Online support at www.pgpyrexia.co.cc

Free monthly AIIMS/ALL INDIA mock exams @ www.pgpyrexia.lefora.com.

Visit today!! Page 4

The following structures are found in the floor of the third ventricle (from anterior to posterior end):

• optic chiasm, • infundibulum, • tuber cinereum, • mammilary bodies, • sub thalamus.

4. Site not affected in posterior cerebral artery infarct is?

A. Midbrain

B. Pons.

C. Thalamus

D. Cortex

Ans: B. Pons

Ref: Harrison

“Two clinical syndromes are commonly observed with occlusion of the PCA: (1) P1 syndrome: midbrain, subthalamic, and thalamic signs, which are due to disease of the proximal P1 segment of the PCA or its penetrating branches (thalamogeniculate, Percheron, and posterior choroidal arteries); and (2) P2 syndrome: cortical temporal and occipital lobe signs, due to occlusion of the P2 segment distal to the junction of the PCA with the posterior communicating artery.”

5. Regarding anterior choroidal artery syndrome, all are true except?

A. Hemipareisis

B. Hemisensory loss,

C. Involvement of anterior limb of internal capsule

D. Homonymous hemianopia

Ans: C. Involvement of anterior limb of internal capsule

Ref: Harrison

“This artery arises from the internal carotid artery and supplies the posterior limb of the internal capsule and the white matter posterolateral to it, through which pass some of the geniculocalcarine fibers . The complete syndrome of anterior choroidal artery occlusion consists of contralateral hemiplegia, hemianesthesia (hypesthesia), and homonymous hemianopia.”

6. Which of the following passes through foramen magnum?

A. Internal Carotid Artery

B. Sympathetic chain

C. Hypoglossal Nerve

D. Vertebral Artery

Ans: D. Vertebral Artery

Online support at www.pgpyrexia.co.cc

Free monthly AIIMS/ALL INDIA mock exams @ www.pgpyrexia.lefora.com.

Visit today!! Page 5

7. Posterior relations of head of pancreas are all except?

A. Common bile duct

B. First part of duodenum

C. Aorta

D. Inferior vena cava

Ans: B. First part of duodenum

Ref: Inderbir singh’s anatomy

“The common bile duct descends in the posterior surface of the head to join the main pancreatic duct at the ampulla of Vater.”-Greenfield’s essential surgery

8. Which is a branch from trunk of brachial plexus?

A. SupraScapular Nerve

B. Long Thoracic Nerve

C. Axillary Nerve

Online support at www.pgpyrexia.co.cc

Free monthly AIIMS/ALL INDIA mock exams @ www.pgpyrexia.lefora.com.

Visit today!! Page 6

D. Nerve to Subclavius Muscle

Ans: A. SupraScapular Nerve and D. Nerve to Subclavius Muscle

9. All of the following are affected in low radial nerve palsy except?

A. Extensor carpi radialis longus

B. Extensor carpi radialis brevis

C. Finger extensors

D. Sensation on dorsum of hand

Ans: A. Extensor carpi radialis longus and D. Sensation on dorsum of hand

Ref: Snell’s anatomy

CONCEPTS:

“Radial Nerve Palsy Radial nerve palsy can be divided into high and low injuries. A high radial nerve palsy involves the radial nerve proper whereas the low palsy involves only the posterior interosseous nerve (PIN)/ Deep Branch of the Radial Nerve”

The deep branch of the radial nerve is a motor nerve to the extensor muscles in the posterior compartment of the forearm. It can be damaged in fractures of the proximal end of the radius or during dislocation of the radial head. The nerve supply to the supinator and the extensor carpi radialis longus will be undamaged, and because the latter muscle is powerful, it will keep the wrist joint extended, and wristdrop will not occur. No sensory loss occurs because this is a motor nerve.

Online support at www.pgpyrexia.co.cc

Free monthly AIIMS/ALL INDIA mock exams @ www.pgpyrexia.lefora.com.

Visit today!! Page 7

10. Which of the following is not supplied by the anterior division of mandibular nerve (V3)?

A. Temporalis

B. Medial pterygoid.

C. Lateral pterygoid

D. Masseter

Ans: B. Medial pterygoid.

Ref: Snell’s anatomy

Branches From the Anterior Division of the Mandibular Nerve

• Masseteric nerve to the masseter muscle • Deep temporal nerves to the temporalis muscle • Nerve to the lateral pterygoid muscle • Buccal nerve to the skin and the mucous membrane of the cheek

(P.S: Nerve to the medial pterygoid muscle is a branch from the Main Trunk of the Mandibular Nerve)

11. Pain sensation from the ethmoid sinus is carried by :

A. Frontal nerve

B. Lacrimal nerve

C. Nasociliary nerve

D. Infraorbital nerve

Ans: C. Nasociliary nerve

Ref: Snell’s anatomy

“Anterior and posterior nerves are branches of nasociliary nerve”

12. All of the following are pneumatic bones except?

A. Frontal

B. Ethmoid

C. Mandible.

D. Maxilla

Ans: C. Mandible [REPEAT]

13. Diaphragm develops from all except:

A. Septum trversum

B. Dorsal mesocardium

C. Pleuroperitoneal membrane

D. Cervical myotomes

Online support at www.pgpyrexia.co.cc

Free monthly AIIMS/ALL INDIA mock exams @ www.pgpyrexia.lefora.com.

Visit today!! Page 8

Ans: D. Cervical myotomes

14. Muscular component of dorsal aorta develops from?

A. Axial mesoderm

B. Paraxial mesoderm

C. Intermediate mesoderm

D. Lateral plate mesoderm

Ans: D. Lateral plate mesoderm

“The lateral plate mesoderm divides to form two layers. Somatic layer develops in association with ectoderm. It forms the body wall and limbs. Visceral layer develops in association with endoderm. It forms the cardiovascular system (including heart), and smooth muscles that surround the gut, blood vessels, and internal organs.”

15. What is the type of joint seen in the growth plate?

A. Fibrous

B. Primary cartilagenous

Online support at www.pgpyrexia.co.cc

Free monthly AIIMS/ALL INDIA mock exams @ www.pgpyrexia.lefora.com.

Visit today!! Page 9

C. Secondary cartilagenous

D. Plane joint

Ans: B. Primary cartilagenous

Ref: Last’s anatomy:

“A primary cartilaginous joint (synchondrosis) is one where bone and hyaline cartilage meet.example is epiphyseal growth plate”

16. Main blood supply of neck of femur?

A. Lateral circumflex femoral

B. Medial circumflex femoral

C. Profunda femoris

D. Popliteal artery

Ans: B. Medial circumflex femoral

Ref: Snell’s Clinical anatomy

Blood Supply to the Femoral Head and Neck Fractures

Anatomic knowledge of the blood supply to the femoral head explains why avascular necrosis of the head can occur after fractures of the neck of the femur. In the young, the epiphysis of the head is supplied by a small branch of the obturator artery, which passes to the head along the ligament of the femoral head.

The upper part of the neck of the femur receives a profuse blood supply from the medial femoral circumflex artery. These branches pierce the capsule and ascend the neck deep to the synovial membrane.

As long as the epiphyseal cartilage remains, no communication occurs between the two sources of blood. In the adult, after the epiphyseal cartilage disappears, an anastomosis between the two sources of blood supply is established. Fractures of the femoral neck interfere with or completely interrupt the blood supply from the root of the femoral neck to the femoral head. The scant blood flow along the small artery that accompanies the round ligament may be insufficient to sustain the viability of the femoral head, and ischemic necrosis gradually takes place.

17. Common carotid artery is palpated at?

A. Upper border of cricoid cartilage

B. Upper border of thyroid cartilage

C. Hyoid bone

D. Cricothyroid membrane

Ans: B. Upper border of thyroid cartilage

Ref: Snell’s anatomy:

“The bifurcation of the common carotid artery into the internal and external carotid arteries can be easily palpated just beneath the anterior border of the sternocleidomastoid muscle at the level of the superior border of the thyroid cartilage. This is a convenient site to take the carotid pulse”

Online support at www.pgpyrexia.co.cc

Free monthly AIIMS/ALL INDIA mock exams @ www.pgpyrexia.lefora.com.

Visit today!! Page 10

18. Urethral crest is situated in:

A. Prostatic urethra

B. Membranous urethra

C. Penile urethra

D. Bulbar urethra

Ans: A. Prostratic urethra

Ref: Snell’s Anatomy

“The prostatic urethra is the widest and most dilatable portion of the entire urethra. On the posterior wall is a longitudinal ridge called the urethral crest”

19. Which among the following is not a component of hypogastric sheath?

A. Broad ligament

B. Transverse cervical ligament

C. lateral ligament

D. Ligament of bladder

Ans: A. Broad ligament

Ref: Gray’s Anatomy:

“The ligamentous extensions of the hypogastric sheath are:

• the lateral ligament of the urinary bladder, • the transverse cervical ligament • Lateral rectal ligament”

PHYSIOLOGY

20. A 32 year old mountaineer has a hematocrit of 70%. What is the possible explanation?

A. Polycythemia with dehydration

B. High altitude cerebral oedema

C. High altitude pulmonary oedema

D. Hemodilution

Ans: A. Polycythemia with dehydration

Ref: Harrison, Cecil’s textbook of medicine

“Polycythemia can be spurious (related to a decrease in plasma volume; Gaisbock's syndrome), primary, or secondary in origin.

The secondary causes are all associated with increases in EPO levels: either a physiologically adapted appropriate elevation based on tissue hypoxia (lung disease, high altitude, CO poisoning, high-affinity hemoglobinopathy) or an abnormal overproduction (renal cysts, renal artery stenosis, tumors with ectopic EPO production).

Online support at www.pgpyrexia.co.cc

Free monthly AIIMS/ALL INDIA mock exams @ www.pgpyrexia.lefora.com.

Visit today!! Page 11

“The most frequent cause of relative polycythemia is dehydration. In settings in which dehydration is likely, fluid balance should be corrected before a hematologic evaluation of an elevated hematocrit is done”

21. A man connected to a body plethysmograph exhales against a closed glottis. What will be the finding?

A. The pressure in both the lungs and the box increases

B. The pressure in both the lungs and the box decreases

C. The pressure in the lungs decreases, but that in the box increases

D. The pressure in the lungs increases, but that in the box decrease

Ans: D. The pressure in the lungs increases, but that in the box decrease

Ref: Chronic obstructive lung diseases By Norbert F. Voelkel, William MacNee

22.Small air way has laminar flow because?

A. Reynold number more than 2000

B. Diameter is very small

C. Velocity very high

D. Total cross sectional area low

Ans: B. Diameter is very small

Ref: Respiratory physiology: a clinical approach By Richard M. Schwartzstein, Michael J. Parker

“In a rapidly branching system such as the lung, fully laminar flow probably only occurs in the small airways because of the very low velocity of air and small radii of individual airways”

23. Orthopnoea in right heart failure develops due to?

A. Reservoir function of pulmonary veins

B. Reservoir function of leg veins

Online support at www.pgpyrexia.co.cc

Free monthly AIIMS/ALL INDIA mock exams @ www.pgpyrexia.lefora.com.

Visit today!! Page 12

C.raised blood pressure

D.not recalled

Ans: A. Reservoir function of pulmonary veins

Ref: Pathophysiology of Heart Disease by Lilly

“Orthopnea results from the redistribution of intravascular blood from the gravity-dependent portions of the body (abdomen and lower extremities) toward the already distended lung vessels after lying down.”(This signifies Reservoir function of pulmonary veins)

24. Compliance is decreased in all except?

A. Pulmonary congestion

B. COPD

C. Decreased surfactant

D. Pulmonary fibrosis

Ans: B. COPD

Ref: Ganong

“Pulmonary fibrosis is a progressive restrictive airway disease of unknown cause in which there is stiffening and scarring of the lung. The curve is shifted upward and to the left (compliance is increased) in emphysema. It should be noted that compliance is a static measure of lung and chest recoil.”

25. Deoxygenated blood is not seen in

A. Pulmonary artery

B. Umbilical artery

C. Umbilical vein

D. Renal vein

Ans: c. Umbilical vein

(Easy question,so no explanation provided!!)

26. Main site of water absorption is:

A. Jejunum

B. Colon

C. Ileum

D. Stomach

Ans: A. Jejunum [REPEAT from AIIMS may 2010]

27. All are true about blood coagulation except?

A. Factor 10 in a part of both intrinsic and extrinsic pathway

Online support at www.pgpyrexia.co.cc

Free monthly AIIMS/ALL INDIA mock exams @ www.pgpyrexia.lefora.com.

Visit today!! Page 13

B. Extrinsic pathway is activated by contact with plasma and negatively charged proteins

C. Calcium is very important for coagulation

D. Intrinsic pathway can be activated in vitro.

Ans: B. Extrinsic pathway is activated by contact with plasma and negatively charged proteins

Ref: Ganong 23rd edition

“Factor X can be activated by either of two systems, known as intrinsic and extrinsic (option A). The initial reaction in the intrinsic system is conversion of inactive factor XII to active factor XII (XIIa) can be brought about in vitro (option D)by exposing the blood to glass, or in vivo by collagen fibers underlying the endothelium. The extrinsic system is triggered by the release of tissue thromboplastin, a protein–phospholipid mixture that activates factor VII. Tissue thromboplastin and factor VII activate factors IX and X. In the presence of PL, Ca2+(option C), and factor V, activated factor X catalyzes the conversion of prothrombin to thrombin. The extrinsic pathway is inhibited by a tissue factor pathway inhibitor”

28. Mineralocorticoid receptor is not present in?

A. Liver

B. Colon

C. Hippocampus

D. Kidney

Ans: A. Liver [REPEAT from AIIMS may 2010]

FORENSIC

29. Child brought to casualty with reports of violent shaking by parents. Most likely injury is?

A. Long bone fracture

B. Ruptured spleen

C. Subdural hematoma

D. Skull bone fracture

Ans: C. Subdural hematoma

Ref: Myers on Evidence in Child, Domestic and Elder Abuse Cases, Volume 1 By John E. B. Myers

This is a case of shaking baby syndrome.Violent shaking can shear the the “bridging” veins in exterior of brain and can result in subdural haematoma.

30. Dental numbering is done by all except?

A. FDI two digit system

B. Anatomic and diagramatic charting

C. Palmer notation

D. Not recalled

Ans: (?) most likely fourth option was the answer since the other three are used

There three different numbering systems used to to identify teeth in in dentistry:

Online support at www.pgpyrexia.co.cc

Free monthly AIIMS/ALL INDIA mock exams @ www.pgpyrexia.lefora.com.

Visit today!! Page 14

1.The Universal Numbering System has been adopted the the ADA and isuse use most general dentists today.

2. The Palmer Notation Numbering System used by some orthodontists

3. The Federation Dentaire Internationale Numbering System (FDI). Internationally

31. Auto-Rickshaw ran over a child’s thigh, there is a mark of the tyre tracks, it is an example of?

A. Contact bruise

B. Patterned bruise

C. Imprint abrasion.

D. Ectopic bruise

Ans: C. Imprint abrasion [REPEAT]

32. Sparrow marks on the face are seen in?

A. Gunshot injuries

B. Stab injury of face

C. Vitriolage

D. Windshield glass injury

Ans: D. Windshield glass injury

Sparrow’s foot marks are bizarre shaped lacerations that result from the face coming in contact with shattered windshield glass

It is a commonly seen in front seat passengers in road traffic accidents

33. Signature fracture refers to?

A. Depressed skull fracture

B. Suture displacement fracture

C. Contrecoup injury

D. Fracture at foramen magnum

Ans: A. Depressed skull fracture

Depressed Fracture: It is due to direct impact of weapon on the skull where bone is depressed depending upon the force applied. Since the depression may resemble the weapon, this fracture is also called 'fracture signature'

34. Rave drug is?

A. Cannabis

B. Cocaine

C. Heroin

D. Ecstasy

Online support at www.pgpyrexia.co.cc

Free monthly AIIMS/ALL INDIA mock exams @ www.pgpyrexia.lefora.com.

Visit today!! Page 15

Ans: D. Ecstasy

35. Gun powder on clothing can be visualized by?

A. Magnifying lens

B. UV rays

C. Infrared rays

D. Liquid spray/ solution spray

Ans: C. Infrared rays

Ref: Textbook of Forensic Medicine and Toxicology by Nagesh Kumar Rao

36. A patient presented to the casuality with bluish pigmentation of conjunctiva, mucous membranes, nails and tachycardia after ingestion of a poison. What is the poison:

A. Mercury

B. Arsenic

C. Lead

D. Silver

Ans: D. Silver

Argyria is a condition caused by improper exposure to chemical forms of the element silver, silver dust, or silver compounds. Chronic intake of silver products can result in an accumulation of silver or silver sulfide particles in the skin. As in photography (where silver is used due to its reactivity with light), these particles in the skin darken with exposure to sunlight, resulting in a blue or gray discoloration of the skin

37. Which among the following does not cause hyperpyrexia?

A. MAOI

B. Alcohol

C. Cresol

D. Amphetamine

Ans: B. Alcohol

Medications and alcohol are additional factors that can predispose to hypothermia by suppressing shivering or inducing vasodilation (eg, alcohol, benzodiazepines, cyclic antidepressants)

38. A poison which is illuminous, trlucent and waxy?

Online support at www.pgpyrexia.co.cc

Free monthly AIIMS/ALL INDIA mock exams @ www.pgpyrexia.lefora.com.

Visit today!! Page 16

A. Iodine

B. Ammonium bromide

C. Cobra venom

D. Yellow phosphorous

Ans: D. Yellow phosphorous

Quick points:

White P: a translucent, waxy white solid.

Red P: a bright red powder.

Black P: a very dark red/black crystalline mass

MICROBIOLOGY

39. False regarding Japanese encephalitis is:

A. Epidemic is 2-3 cases in a village

B. Mosquito bite is always associated with disease

C. 70% affected in infants

D. Apparent and nonapparent ratio 1:100

Ans: B. Mosquito bite is always associated with disease

High yield points: (Ref:Nelson)

• Estimates of the ratio of symptomatic disease to asymptomatic infection vary between 1 in 25 and 1 in 1000 • The annual incidence in endemic areas ranges from 1–10/10,000 population. Children <15 yr of age are

principally affected(explains option c), with nearly universal exposure by adulthood.

40. A sewer worker presented with fever. Lab findings revealed renal failure with increased BUN and serum creatinine. What is the most appropriate drug to give him?

A. Cotrimoxazole

B. Erythromycin

C. Ciprofloxacin

D. Benzyl penicillin

Ans: D.Benzyl penicillin

This is a case of Leptospirosis for which penicillin is first choice drug

41. About yaws all are true except?

A. Caused by Treponema pertenue

B. Transmitted non-venerally

Online support at www.pgpyrexia.co.cc

Free monthly AIIMS/ALL INDIA mock exams @ www.pgpyrexia.lefora.com.

Visit today!! Page 17

C. Secondary yaws can involve bones

D. Last stages involve heart and nerves

Ans: D. Last stages involve heart and nerves

Ref: Harrison:

Also known as pian, framboesia, or bouba, yaws is caused by T. pallidum subspecies pertenue and is characterized by the development of one or several primary lesions ("mother yaw"), which is followed by the appearance of multiple disseminated skin lesions

Painful papillomatous lesions on the soles of the feet result in a painful crablike gait ("crab yaws"), and periostitis may result in nocturnal bone pain and polydactylitis. Late yaws is manifested by gummas of the skin and long bone, hyperkeratoses of the palms and soles, osteitis and periostitis, and hydrarthrosis. The late gummatous lesions are characteristically extensive. Destruction of the nose, maxilla, palate, and pharynx is termed gangosa and is similar to the destructive lesions seen in leprosy and leishmaniasis.

42. Aflatoxin is produced by?

A. Aspergillus flavus

B. Aspergillus niger

C. Candida

D. Dermatophytes

Ans: A. Aspergillus flavus

Ref: Jawetz Microbiology

One of the most potent is aflatoxin, which is elaborated by Aspergillus flavus and related molds and is a frequent contaminant of peanuts, corn, grains, and other foods.

43. False about C.diphtheriae is?

A. Toxin production is chromosome mediated

B. Virulence is due to toxin production

C. Toxic to heart and neurons

D. Toxin blocks protein synthesis

Ans: A. Toxin production is chromosome mediated (it is plasmid mediated)

“Only those strains of Corynebacterium diphtheriae that are lysogenized by a specific Beta phage produce diphtheria toxin.”

44. Capsular virulence is seen in all except?

A. Neisseria meningitidis

B. Pneumococcus

C. Bordetella pertussis

D. Haemophilus influenza

Online support at www.pgpyrexia.co.cc

Free monthly AIIMS/ALL INDIA mock exams @ www.pgpyrexia.lefora.com.

Visit today!! Page 18

Ans: C. Bordetella pertussis

The organism's most important virulence factor is pertussis toxin, which is composed of a B oligomer–binding subunit and an enzymatically active A protomer that ADP-ribosylates a guanine nucleotide-binding regulatory protein (G protein) in target cells, producing a variety of biologic effects.

45. Carrier state is not important in transmission of:

A. Measles

B. Typhoid

C. Polio

D. Diphtheria

Ans: A. Measles [REPEAT]

46. Visceral larva migrans is seen in?

A. Strongyloides

B. Ancylostoma

C. Toxocara canis

D. Visceral leishmaniasis

Ans: C. Toxocara canis

Visceral larva migrans is caused by worms (parasites) that infect the intestines of dogs and cats. The dog parasite is called Toxocara canis and the cat parasite is called Toxocara cati.

47. Cavitation is seen in?

A. Mycolplasma pneumonia

B. Tuberculous pneumonia

C. Streptococcal pneumonia

D. Staphylococcus pneumonia.

Ans: D.Staphylococcus pneumonia [REPEAT]

48. A patient comes with history of fever and cough unresponsive to antibiotics.It was partially Acid fast. X-ray shows consolidation. Bronchioalveolar lavage shows gram positive branching filaments. What is the diagnosis?

A. Actinomycosis

B. Nocardiosis

C. Aspergillus

D.mycoplasma

Ans: B. Nocardiosis

Ref: Harrison

Online support at www.pgpyrexia.co.cc

Free monthly AIIMS/ALL INDIA mock exams @ www.pgpyrexia.lefora.com.

Visit today!! Page 19

“Pneumonia, the most common form of nocardial disease in the respiratory tract, is typically subacute; symptoms have usually been present for days or weeks at presentation. The onset is occasionally more acute in immunosuppressed patients. Cough is prominent and produces small amounts of thick, purulent sputum that is not malodorous. The first step in diagnosis is examination of sputum or pus for crooked, branching, beaded, gram-positive filaments 1 microm wide and up to 50microm long . Most nocardiae are acid-fast in direct smears if a weak acid is used for decolorization

49. A farmer developed a swelling in the inguinal region which later ulcerated. What stain can be used to detect bipolar stained organisms?

A. Albert's stain

B. Waysons stain

C. Ziehl neelsen stain

D. Nigrosin stain

Ans: B. Waysons stain [REPEAT]

50. A patient presents with signs of pneumonia. The bacterium obtained from sputum grows on sheep agar. What test is used to identify the type of organism?

A. Bile solubility

B. Bacitracin sensitivity

C. Coagulase test

D. Gram staining

Ans: A. Bile solubility [REPEAT]

51. 'C' in C reactive protein stands for:

A. Capsular polysaccharide in pneumococcus

B. Concanavalin-a

C. Calretinin

D. Cellular

Ans: A. Capsular polysaccharide in pneumococcus [REPEAT]

52. False about pneumococcus is?

A. Capsule aids in virulence

B. Commonest cause of otitis media and pneumonia

C. Meningitis caused by it is milder than other organisms

D.not recalled

Ans: C. Meningitis caused by it is milder than other organisms [REPEAT]

53. A female presents with sings of meningitis. CSF shows gram positive bacilli. It is most probably?

Online support at www.pgpyrexia.co.cc

Free monthly AIIMS/ALL INDIA mock exams @ www.pgpyrexia.lefora.com.

Visit today!! Page 20

A. Listeria

B. Haemophilus influenzae

C. Pneumococcus

D.Staphylococcus

Ans: A. Listeria [REPEAT]

54. All are true about parvovirus b19 except?

A. <10 % spread by trplacental route

B. Spread by respiratory route

C. It is a DNA virus

D. Affects erythroid progenitor cells

Ans: A. <10 % spread by trplacental route [REPEAT]

55.All are transmitted transplacentally except?

A. Toxoplasmosis

B. HSV

C. CMV

D. Parvo virus

Ans: B. HSV [REPEAT]

56. True regarding chlamydia trachomatis is?

A. Culture of purulent endocervical discharge is used for isolation of organism

B. Patient using OCP's are carriers

C. Not recalled

D. Not recalled

Ans: B. Patient using OCP's are carriers

Ref: BEREK and NOVAK Gynecology

Chlamydial colonization of the cervix appears more likely in OC users than in nonusers but, despite this finding, several case control studies have found a reduced risk of acute pelvic inflammatory disease among OC users (163,164). In contrast, a recent study found no protection with OC use (165).*****OPTION B

The diagnosis of cervicitis is based on the finding of a purulent endocervical discharge, generally yellow or green in color and referred to as “mucopus” (it is not strictly purulent)

57. True regarding leptospirosis is?

A. Rats are the most important reservoirs

Online support at www.pgpyrexia.co.cc

Free monthly AIIMS/ALL INDIA mock exams @ www.pgpyrexia.lefora.com.

Visit today!! Page 21

B. Fluroquinolones are the DOC

C. Person to person trmission

D. Hepatorenal syndrome occurs in 50% cases

Ans: A. Rats are the most important reservoirs

Ref: Harrison 17th ed

Rodents, especially rats, are the most important reservoir, although other wild mammals as well as domestic and farm animals may also harbor leptospires. These microorganisms establish a symbiotic relationship with their host and can persist in the renal tubules for years.

58. A 7 month old child presents with complaints of recurrent bouts of whooping cough. What is the best specimen to confirm the diagnosis?

A. Nasophayngeal swab

B. Cough plate culture

C. Tracheal aspirate

D. Blood culture

Ans: A. Nasopharyngeal swab

Ref: Harrison

Culture of nasopharyngeal secretions remains the gold standard of diagnosis, although DNA detection by polymerase chain reaction (PCR) is replacing culture in many laboratories because of increased sensitivity and quicker results. The best specimen is collected by nasopharyngeal aspiration, in which a fine flexible plastic catheter attached to a 10-mL syringe is passed into the nasopharynx and withdrawn while gentle suction is applied.

59. Tetracycline is used in the prophylaxis of which of the following diseases?

A. Cholera

B. Brucellosis

C. Leptospirosis

D. Meningitis

Ans: A. Cholera

Ref: Goodman & Gilman's Pharmacology

Tetracycline is used in chemoprophylaxis of Cholera.

Doxycycline is used in chemoprophylaxis of Leptospirosis.

60. Regarding Clostridium tetani, all are true except?

A. Spores are resistant to heat

B. 3 doses give immunity in primary immunisation

C. Incubation period is 6-10 days

D. Person to person transmission does not occur

Online support at www.pgpyrexia.co.cc

Free monthly AIIMS/ALL INDIA mock exams @ www.pgpyrexia.lefora.com.

Visit today!! Page 22

Ans: B. 3 doses give immunity in primary immunization

61. Campylobacter jejuni false is:

A. Commonest cause of campylobacteriosis

B. Polutry source of infection

C. Humans are only reservoir

D. Associated with GBS

Ans: C. Humans are the only reservoir [REPEAT AIIMS09]

62. Which among the following is the most common fungal infection seen in immuno competent patients?

A. Aspergillus

B. Candida

C. Cryptococcus

D. Mucor

Ans: B. Candida

Ref: www.fungalforum.com/Presentations/Moskou.ppt

Invasive fungal infections in immunocompetent patients

Contributing factors to the development of a fungal infection

• Increased exposure to air, water and food contaminated with fungal spores • Virulence of the fungus • Host immune status

• Underlying disease • Immunosuppressive therapy

Invasive fungal infections :

Candida Endogenous (mostly)

Aspergillus Exogenous

Cryptococcus

Zygomycetes

Rare filamentous species

63. Most common cause of meningoencephalitis in children?

A. HSV

B. Enterovirus

C. Mumps

Online support at www.pgpyrexia.co.cc

Free monthly AIIMS/ALL INDIA mock exams @ www.pgpyrexia.lefora.com.

Visit today!! Page 23

D. Listeria

Ans: B. Enterovirus [REPEAT]

64. 8 yr old with sign of meningitis and ear infections with ring enhancement in the fronto temporal lobe all can cause it except?

A. H.infuenzeae

B. Staphylococcus

C. Pneumococcus

D. Pseudomonas

Ans: A. H.influenzae

Ref: ENT Dhingra

This is a case of cerebral abcess secondary to otitis media Dhingra mentions B/C/D as common causes…and mentions H.influenzae as RARELY SEEN.

PHARMACOLOGY:

65. Prolonged treatment with INH leads to deficiency of?

A. Pyridoxine

B. Thiamine

C. Pantothenic acid

D. Niacin

ANS: A. Pyridoxine

Ref: Katzung

“Peripheral neuropathy is more likely to occur in slow acetylators and patients with predisposing conditions such as malnutrition, alcoholism, diabetes, AIDS, and uremia. Neuropathy is due to a relative pyridoxine deficiency. Isoniazid promotes excretion of pyridoxine, and this toxicity is readily reversed by administration of pyridoxine in a dosage as low as 10 mg/d.”

66. Integrase inhibitor approved for treatment of HIV is?

A. Raltegravir

B. Indinavir

C. Lopinavir

D. Elvitegravir

Ans: A. Raltegravir

Raltegravir is an HIV integrase strand transfer inhibitor that blocks the action of HIV integrase

Online support at www.pgpyrexia.co.cc

Free monthly AIIMS/ALL INDIA mock exams @ www.pgpyrexia.lefora.com.

Visit today!! Page 24

67. All of the following are true about erlotinib except?

A. Tyrosine kinase inhibitor

B. Food delays its absorption

C. Rashes can occur

D. Used in non small cell lung cancer when there is no response to other chemotherapeutic agents

Ans: B. Food delays its absorption

Ref: Katzung:

“Gefitinib (ZD1694; Iressa) and erlotinib (OSI-774; Tarceva) are small molecule inhibitors of the tyrosine kinase domain associated with the EGFR, and both are approved for use in the treatment of non-small cell lung cancer that is refractory to at least one prior chemotherapy regimen.Patients who are nonsmokers and who have a bronchoalveolar histologic subtype appear to be more responsive to these agents. With respect to toxicity, an acneiform skin rash and diarrhea are the most common adverse effects observed with these small molecules.

68. All are true regarding selective estrogen receptor downregulator (SERD),fulvestrant except?

A. Used for breast cancer

B. Is a selective oestrogen antagonist

C. Is slower acting, safer, LESS effective than SERM

D. Given as once a month in dose

Ans: C. Is slower acting, safer, LESS effective than SERM

Ref: Goodman&Gilman

“Fulvestrant is the first FDA approved agent in the new class of estrogen-receptor downregulators, which were hypothesized to have an improved safety profile, faster onset, and longer duration of action than the SERMs due to their pure ER antagonist activity .

Fulvestrant was approved in 2002 for postmenopausal women with hormone receptor-positive metastatic breast cancer that has progressed despite antiestrogen therapy.

Fulvestrant typically is administered as a 250-mg intramuscular injection at monthly intervals”

69. All are used in the treatment of hot flushes except?

A. Tamoxifene

B. Venlafaxine

C. Estrogen

D. Not recalled

Ans: A. Tamoxifen [REPEAT]

70. Which of the following is true?

A. Acetylcholinesterase inhibition by Malathion can be reversed by increasing the level of acetylcholinesterase

B. Sulphonilamide inhibits folate reductase irrevesibly

Online support at www.pgpyrexia.co.cc

Free monthly AIIMS/ALL INDIA mock exams @ www.pgpyrexia.lefora.com.

Visit today!! Page 25

C. Flouoroacetate competetively inhibits aconitase

D. Ethanol inhibits aldehyde dehydrogenase when used in methanol poisoning

Ans: None

Please read below:

Option A:

Parathion and malathion are thiophosphate (sulfur-containing phosphate) prodrugs that are inactive as such; they are converted to the phosphate derivatives in animals and plants and are used as insecticides.

Atropine is only for muscarinic symptoms.. not for the anticholinesterase effects in OPC poisoning ...we all know that

[Ref: Harrison]

Sulfonamide inhibits Folate reductase in a reversible manner.

Also, fluoroacetate is a suicide inhibitor which inhibits irreversibly but non competitively

Ethanol inhibits ALCOHOL DEHYDROGENASE when used in METHANOL poisoning

71. Amphotericin B causes deficiency of?

A. Na

B. Ca

C. K

D. Mg

Ans: C. K+

Reference: Goodman and Gilman Pharmacology

Renal tubular acidosis and renal wasting of K+ and Mg2+ also may be seen during and for several weeks after therapy. Supplemental K+ is required in one-third of patients on prolonged therapy.

72. Which is not given to treat diabetic macular edema?

A. Laser Photocoagulation

B. Acetazolamide

C. Tamoxifen

D. None

Ans: C. Tamoxifen

We all know that laser Photocoagulation is a standard treatment for diabetic macular edema.

Macular Edema: A Practical Approach By G. Coscas states that “Daily oral acetazolamide decreases the central macular thickness and allows CME regression that can be seen on OCT imaging”

73. Low dose oral contraceptive pill contains?

A. Levonorgestrel

Online support at www.pgpyrexia.co.cc

Free monthly AIIMS/ALL INDIA mock exams @ www.pgpyrexia.lefora.com.

Visit today!! Page 26

B. Norgestrel

C. Desogestrel

D. Norethihisterone

Ans: C. Desogestrel (?), A.Levonorgestrol (?)

Caution:

LOW DOSE OCP

All the options can be used as low dose ocp

Any progesterone in combination with less than 0.05 mg of estrogen is low dose.

Most common in India is mala-d which uses LNG.

74. Which among the following is preferred in a patient with decreased renal functio to avoid contrast nephropathy?

A. N-acetylcysteine

B. Fenoldopam

C. Low osmolar contrast

D. Mannitol

Ans: C. Low osmolar contrast

Ref: Contrast Media: Safety Issues and ESUR Guidelines By Henrik S. Thomsen, Judith A. W. Webb

The type of contrast medium used is an important risk factor for the development of contrast medium- induced nephropathy with iso-osmolar contrast media (IOCM) and low-osmolar contrast media (LOCM) being less nephrotoxic than high osmolar agents in patients with pre-existing renal impairment

75. Pregnant mother at 35 weeks of gestation. What drug can you not give her for treatment of SLE?

A. Prednisolone

B. Methotrexate

C. Sulfsalazine

D. Hydroxychloroquine

Ans: B. Methotrexate [REPEAT]

76. A primigravida in 1st trimester had sputum positive for acid fast bacillus. What is the preferred treatment?

A. Treatment deferred till 2nd trimester

B. Category 1 DOTS

C. Category 2 DOTS

D. Category 3 DOTS

Ans: B. Category 1 DOTS [REPEAT]

Online support at www.pgpyrexia.co.cc

Free monthly AIIMS/ALL INDIA mock exams @ www.pgpyrexia.lefora.com.

Visit today!! Page 27

77. Drug of choice for central Diabetes Insipidus is?

A. Desmopressin

B. Leuperolide

C. Thiazide diuretics

D. Lithium

Ans: A. Desmopressin

Ref: Harrison

The signs and symptoms of uncomplicated pituitary DI can be eliminated completely by treatment with desmopressin (DDAVP), a synthetic analogue of AVP .

It acts selectively at V2 receptors to increase urine concentration and decrease urine flow in a dose-dependent manner .

78. Which of the following antihypertensive drugs is contraindicated in a patient on Lithium in order to prevent toxicity?

A. Clonidine

B. Beta blockers

C. Calcium channel blockers

D. Diuretics.

Ans: D. Diuretics

Ref: Goodman and Gilman Pharmacology

Most of the renal tubular reabsorption of Li+ occurs in the proximal tubule. Nevertheless, Li+ retention can be increased by any diuretic that leads to depletion of Na+, particularly the thiazides .

79. Administration of which of the following drug needs alkalization of urine?

A. Cytosine arabinoside

B. Methotrexate

C. Cisplatin

D. Ifosfamide

Ans:A Methotrexate

Goldfrank's toxicologic emergencies

By Lewis R. Goldfrank, Neal Flomenbaum

Urinary alkalinisation with Sodium Bicarbonate is routinely used during high dose Methotrexate cancer chemotherapy.

80. Pulmonary toxicity is seen with?

A. Bleomycin

B. Cisplatin

Online support at www.pgpyrexia.co.cc

Free monthly AIIMS/ALL INDIA mock exams @ www.pgpyrexia.lefora.com.

Visit today!! Page 28

C. Methotrexate

D. Actinomycin D

Ans: A. Bleomycin

Ref: Goodman and Gilman pharmacology

The most serious adverse reaction to bleomycin is pulmonary toxicity, which begins with a dry cough, fine rales, and diffuse basilar infiltrates on x-ray and may progress to life-threatening pulmonary fibrosis. Radiologic changes may be indistinguishable from interstitial infection or tumor, but may progress to dense fibrosis, cavitation, atelectasis or lobar collapse, or even apparent consolidation.

81. Medical treatment for variceal bleed is by?

A. Octreotide

B. Pantaprazole

C. Desmopressin

D. Dopamine

Ans: A. Octreotide

Reference: Harrison Principles of Internal Medicine 17th ed

The medical management of acute variceal hemorrhage includes the use of vasoconstricting agents, usually somatostatin or Octreotide. Vasopressin was used in the past but is no longer commonly used.

82. An 8 year old boy completed 8 out of 10 day course of cefaclor. Now he developed a generalized erythmatic rash which is mildly pruritic and lymphadenopathy. Diagnosis is?

A. Kawasaki disease

B. Type 3 hypersensitivity

C. Anaphylaxis

D. Infectious mononucleosis

Ans: B. Type 3 hypersensitivity [REPEAT]

83. Tolerance in opioids develops to all except?

A. Miosis

B. Analgesia

C. Euphoria

D. Nausea and vomiting

Ans: A. Miosis

Ref: KDT

Tolerance develops to all actions of opioid except miosis and constipation

Online support at www.pgpyrexia.co.cc

Free monthly AIIMS/ALL INDIA mock exams @ www.pgpyrexia.lefora.com.

Visit today!! Page 29

84. Maintenance dose of which of the following drugs is used worldwide for opioid dependence?

A. Naltrexone

B. Methadone

C. lmma

D. Disulfiram

Ans: B. Methadone [REPEAT]

85. Which among the following is not used to treat alcohol dependence?

A. Flumazenil

B. Acamprosate

C. Naltrexone

D. Disulfiram

Ans: A. Flumazenil [REPEAT]

86. Antidepressant drug that can be used in nocturnal eneuresis?

A. Imipramine

B. Fluoxamine

C. Doxepin

D.Amoxapine

Ans: A. Imipramine

Ref: Kaplan & Sadock's Synopsis of Psychiatry

Imipramine is efficacious and has been approved for use in treating childhood enuresis, primarily on a short-term basis. Initially, up to 30 percent of patients with enuresis stay dry, and up to 85 percent wet less frequently than before treatment.

The success often does not last, however, and tolerance can develop after 6 weeks of therapy. Once the drug is discontinued, relapse and enuresis at former frequencies usually occur within a few months. The drug's adverse effects, which include cardiotoxicity, are also a serious problem.

87. All are actions of muscarinic antagonist except?

A. Decreases gastric secretion

B. Prolongs a-v conduction

C. Decreases respiratory secretions

D. Contraction of radial muscles of iris

Ans: D. Contraction of radial muscles of iris

On seeing the Q first it appears as if all 4 options are correct....Anyway cholinergic system contracts circular muscle of iris...an Antimuscarinic agen will block this and this inturn leads to pupilary dilatation....contraction of radial muscles of Iris is a function of sympathetic system.

Online support at www.pgpyrexia.co.cc

Free monthly AIIMS/ALL INDIA mock exams @ www.pgpyrexia.lefora.com.

Visit today!! Page 30

88. Serotonin syndrome – true is all except?

A. Dantrolene given in treatment

B. Characterised by increased temperature and hypertension

C. SSRI can precipitate it

D. Usually predictable and not idiosyncratic

Ans: A. Dantrolene given in treatment

“Usually serotonin antagonists like cyproheptadine,methysergide are used in the treatment”

89. A patient who was given primaquine develops hemolysis. Diagnosis is

A. Glucose 6 phosphate dehydrogenase deficiency

B. Glucose 6 phosphatase deficiency

C. Pyruvate kinase deficiency

D. HMG CoA reductase deficiency

Ans: A. Glucose 6 phosphate dehydrogenase deficiency [REPEAT]

90. What will you give to treat hypothyroidism in a patient with ischemic heart disease?

A. Low dose of levothyroxine

B. Normal dose of levothyroxine

C. Do not give levothyroxine

D. Thyroid extract

Ans: A. Low dose of levothyroxine

The thyroid and cardiovascular risk: Merck European Thyroid Symposium ... By Karl-Michael Derwahl, Leonides Duntas, Sigrid Butz

“The optimal replacement dose should take into account the age of the patient, the cause of hypothyroidism and the presence of ischemic heart disease. Treatment of subclinical hypothyroidism is recommended in presence of TSH levels more than 10 MU/mL.

In patients with long-standing hypothyroidism and coexisting ischaemic heart disease, graded introduction of replacement therapy (starting with low dose)over several weeks is frequently used.

91. Which of the following contraception method is contraindicated in women with epilepsy?

A. Oral Contraceptive pill

B. IUCD

C. Condom

D. Mifepristone

Ans: A. Oral Contraceptive pill

Online support at www.pgpyrexia.co.cc

Free monthly AIIMS/ALL INDIA mock exams @ www.pgpyrexia.lefora.com.

Visit today!! Page 31

“OCP should not be used in a known case of epilepsy”-Park

92.Which of the following is not a side effect of methotrexate therapy?

A.Diarrhoea

B. Bone marrow suppression

C. Mucositis

D. Pancreatitis

Ans: D. Pancreatitis

Ref: Katzung:

Toxicities of methotrexate:

• Mucositis, • diarrhea, • bone marrow depression with leukopenia and thrombocytopenia

93. All are true about meglitinides except?

A. Decreases post parandial hyperglycemia

B. Hypoglycemia less common than sulfonylureas

C. It decreases insulin resistance

D. It acts by releasing insulin

Ans: C. It decreases Insulin resistance

Ref: Katzung

The meglitinides are a relatively new class of insulin secretagogues, Which modulate B-cell insulin release by regulating potassium efflux through the potassium channels.

Because of its rapid onset, repaglinide is indicated for use in controlling postprandial glucose excursions.

The incidence of hypoglycemia may be the lowest of all the secretagogues, and ans:it has the advantage of being safe in individuals with very reduced renal function.

94. Which among the following is the best inotrope drug for use in right heart failure with primary pulmonary hypertension?

A. Dobutamine

B. Digoxin

C. Halothane

D. Milrinone

Ans: D. Milrinone

Ref: Goodman and Gilman Pharmacology

Online support at www.pgpyrexia.co.cc

Free monthly AIIMS/ALL INDIA mock exams @ www.pgpyrexia.lefora.com.

Visit today!! Page 32

Milrinone cause direct stimulation of myocardial contractility and acceleration of myocardial relaxation. In addition, they cause balanced arterial and venous dilation with a consequent fall in systemic and pulmonary vascular resistances, and left and right heart filling pressures.

The arterial and venous dilator effects of milrinone are greater than those of dobutamine at doses that produce comparable increases in cardiac output

95. Which is not seen in digoxin toxicity?

A. Biventricular tachycardia

B. Atrial tachycardia with variable AV block

C. Ventricular bigeminy

D. Regularisation of AF

Ans: D. Regularisation of AF

Ref: Goodman & Gilman

“Among the more common electrophysiological manifestations are ectopic beats of AV junctional or ventricular origin, first-degree AV block, an excessively slow ventricular rateresponse to atrial fibrillation, or an accelerated AV junctional pacemaker. These often require only a dosage adjustment and appropriate monitoring. Sinus bradycardia, sinoatrial arrest or exit block,and second- or third-degree AV conduction delay usually respond to atropine, although temporary ventricular pacing may be necessary. Potassium administration should be considered for patients with evidence of increased AV junctional or ventricular automaticity, even when the serum K+ is in the normal range, unless high-grade AV block also is present.

96. Which of the following is given to treat thrombocytopenia secondary to myelosuppresive therapy?

A. Filgrastim

B. Oprelvekin

C. Erythropoietin

D. Sargramostim

Ans: B. Oprelvekin

“Oprelvekin (Recombinant Human Interleukin-11) is used to treat or prevent thrombocytopenia associated with chemotherapy for solid tumor cancer treatment.

97. All are true about APREPITANT except?

A. Agonist at NK1

B. Crosses blood brain barrier

C. Ameliorates nausea and vomiting of chemotherapy

D. Metabolized by CYP450

Ans: A. Agonist at NK1 Receptor

Ref: Katzung Pharmacology

Online support at www.pgpyrexia.co.cc

Free monthly AIIMS/ALL INDIA mock exams @ www.pgpyrexia.lefora.com.

Visit today!! Page 33

Neurokinin 1 (NK1) receptor antagonists have antiemetic properties that are mediated through central blockade in the area postrema. Aprepitant is a highly selective NK1 receptor antagonist that crosses the blood-brain barrier and occupies brain NK1 receptors. It has no affinity for serotonin, dopamine, or corticosteroid receptors.

98. All are true about ranalozine except?

A. Causes hypotension

B. It is recommended as first line treatment for Angina

C. Improves glycemic control

D. It is an anti anginal drug

Ans: B. It is recommended as a first line treatment of Angina.

Option D is correct- a well known fact

Option A is also correct---It does cause postural hypotension and hypotension

Option C is also correct-- In the CARISA trial, ranolazine improved glycemic control in patients who had diabetes mellitus. In particular, the drug reduced hemoglobin A1C (HbA1C) concentrations by approximately 0.5% in the diabetes subgroup, an effect that persisted during long-term therapy

Now Option B:

Ranolazine was introduced in 2006 as a drug to be used in *****chronic angina***** along with other mainstay drugs….In 2008 it seems to have been given the green signal to be used as a first line agent in ****chronic angina****

This option shall go wrong by 2 ways:

1.Latest editions of standard Cardiology books like Braunwald /Hurst and even Herry all mention it to be used along with other agents..

2.At the present time too the indication is only for CHRONIC angina----the option clearly mentioned ANGINA…..so assuming it to be chronic angina and taking it as a right option when the other three are right ….will be a little too odd….

****So, We feel the answer should be B.Its not a first line management for ANGINA overall though it may be for CHRONIC…****

99. Buprenorphine is?

A. Partial agonist at Mu Receptor

B. Partial agonist at Kappa Receptor

C. Full Agonist at Mu Receptor

D. Antagonist at Kappa receptor

Ans: A. Partial agonist of mu receptor

Reference: Goodman and Gilman

Buprenorphine appears to be a partial mu receptor agonist. Depending on the dose, buprenorphine may cause symptoms of abstinence in patients who have been receiving mu-receptor agonists for several weeks.

100. All of the following decrease bone resorption in osteoporosis except?

A. Alendronate

Online support at www.pgpyrexia.co.cc

Free monthly AIIMS/ALL INDIA mock exams @ www.pgpyrexia.lefora.com.

Visit today!! Page 34

B. Etidronate

C. Strontium

D. Teriparatide.

Ans: D. Teripartide

Reference: Goodman and Gilman Pharmacology

“The mechanism of action of teriparatide is unique in that it possesses anabolic properties and therefore builds bone. (it is not an anti-resorptive)

SPM:

101. Pearson's skewness coefficient is?

A. (Mean-median)/SD

B. Median-mean/SD

C. SD/mean-median

D. SD/median-mean

Ans: A. (Mean-median)/SD

Ref: Statistical methods for health care research - Page 46 by Munro

“Pearson's Skewness Coefficient This measure of skewness is nonalgebraic but is easily calculated and is useful for quick estimates of symmetry. It is defined as: Skewness = (mean - median)/SD”

102. Punnet Square is used for?

A. Genotype of offspring

B. Statistical analysis

C. To measure body surface area

Ans: A. Genotype of offspring

Ref: Concepts Of Genetics, 7/E (With Cd) By Klug

“The genotypes and phenotypes resulting from the recombination of gametes during fertilization can be easily visualized by Punnett’s square.

The Punnett square method is particularly useful when first learning about genetics and how to solve problems”

103. Orthotolidine test is used for detecting:

A. Chlorine

B. Nitrites

C. Nitrates

D. Ammonia

Ans: A. Chlorine [REPEAT]

Online support at www.pgpyrexia.co.cc

Free monthly AIIMS/ALL INDIA mock exams @ www.pgpyrexia.lefora.com.

Visit today!! Page 35

104. All are true except:

A. Human anatomical waste is disposed in a yellow bag

B. Red bag contents can be a source of contamination

C. Black bag is used for incineration ash

D. Blue bag contents are always disposed in secure landfill

Ans: D. Blue bag contents are always disposed in secure landfill

Ref: Park:

• “Blue bag contents can be disposed off by Autoclave/microwave/chemical treatment and destruction shredding”

• It is a Plastic bag/ Puncture proof • It includes waste Cat 4,7

105. Denominator in Maternal Mortality Rate?

A. Total number of live births

B. Total number of married women

C. Total number of births

D. Midyear population

Ans: A. Total number of live births

Ref: Park

“The number of maternal deaths related to childbearing divided by the number of live births in that year”

106. Mother to baby transmission of HIV can be minimised by all except?

A. Zidovudine

B. Vitamin A

C. Vaginal delivery

D. Avoidance of breast feeding

Ans: C. Vaginal delivery [REPEAT]

107. Pasteurised milk is most commonly tested by:

A. Phosphatase test.

B. Coliform test

C. Catalase test

D. Oxidase Test

Ans: A. Phosphatase test [REPEAT]

Online support at www.pgpyrexia.co.cc

Free monthly AIIMS/ALL INDIA mock exams @ www.pgpyrexia.lefora.com.

Visit today!! Page 36

108. Which of following disease is not endemic in India?

A. Rabies

B. Cholera

C. Measles

D. Yellow fever

Ans: D. Yellow fever

“Yellow fever is exotic In India and every precautionary measure is taken to avoid its entry into the country”-Park

109. All are factors responsible for resurgence of malaria except?

A. Drug resistance

B. Use of bed nets

C. Vector resistance

D. Mutation in parasite

Ans: B. Use of bed nets

This is a very important and upcoming topic:

Resurgence of Malaria

Malaria is returning to areas from which it had been eradicated, and spreading in to new areas, such as Central Asia, and Eastern Europe. More people are now dying of malaria than thirty years ago.

There are several factors which contribute to this:

1. Drug resistance is a problem, chloroquine is an extremely safe and cheap drug, but in Asia and an increasing area of Africa and South America the resistance levels are high. In some areas of Asia there is resistance to all the major drugs.

2. Mosquitos are developing resistance to the major classes of insecticide which have been used to control the disease.

3. Population and demographic changes have resulted in more people moving into densely populated areas, thereby increasing transmission.

4. Human environmental changes such as road building, mining, deforestation, and new agricultural and irrigation projects have created new breeding sites.

5. Migration, climatic change and the creation of new habitats have all resulted in people who have no natural immunity to the disease being exposed. This results in much higher rates of disease and death.

Online support at www.pgpyrexia.co.cc

Free monthly AIIMS/ALL INDIA mock exams @ www.pgpyrexia.lefora.com.

Visit today!! Page 37

6. In many regions, malaria control programs have deteriorated or been abandoned.

110. Which insect among the following is not resistant to DDT?

A. Musca domestica

B. Phlebotomus

C. Culex

D. Anopheles stephensi

Ans: B. Phlebotomus

Ref: Insecticide resistance in arthropods by R.Pal

“the only genera of medical importance in which resistance has not yet been confirmed are Phlebotomus sandflies, Glossina tsetseflies, and reduviid bugs”

111. Which of the following does not cause indoor air pollution?

A. CO

B. Nitrogen dioxide

C. Radon

D. Mercury vapor

Ans: D. Mercury vapor

Refrigeration & air conditioning technology By William C. Whitman, William M. Johnson, John A. Tomczyk

-Radon is the most important source:

-CO is a product of combustion that causes indoor air pollution

-Among different nitrogen oxides,nitrogen dioxide is most widely considered in indoor air pollution

112. A graph of Normal blood sugar level curve and Diabetic blood sugar level curve was shown. An area was seen overlapping towards the normal gycemic curve. A point at 120 mg/dl was shown too. Question : What does that area represent?

A. True positive

B. False positive

C. True negative

D. False negative

Ans: B. False positive

If the 'area' corrresponds to ppl with blood sugars > 120 (if tat is wat overlapping towards normal curve means) but still not diabetics, it indicates false positives.. ans is B.

113. investigator finds out that 5 independent factors influence the occurrence of a disease. Comprision of multiple factors responsible for a disease can be assessed by?

A. ANOVA

Online support at www.pgpyrexia.co.cc

Free monthly AIIMS/ALL INDIA mock exams @ www.pgpyrexia.lefora.com.

Visit today!! Page 38

B. Multiple linear regression

C. Chi square

D. Multiple logistic regression

Ans: B. Multiple linear regression

“Multiple linear regression attempts to model the relationship between two or more explanatory variables and a response variable by fitting a linear equation to observed data. Every value of the independent variable x is associated with a value of the dependent variable y”

114. Not a part of national screening program?

A. Diabetes mellitus

B. Dental caries

C. Refractive error

D. Carcinoma cervix

Ans: B. Dental caries

-School screening for refractive error comes under NPCB

-Carcinoma cervix is well known

-National Screening Programme for Diabetic Retinopathy Screening aims to reduce blindness caused by diabetes (asked in Nov AIIMS ’10)

115. McKeon's theory on reduced prevalence of TB?

A. Increased awareness and knowledge

B. Medical advancement

C. Behavioural modification

D. Social and environmental factor

Ans: D. Social and environmental factor

Reference: “ McKeown and the Idea That Social Conditions Are Fundamental Causes of Disease Bruce G. Link, PhD and Jo C. Phelan, PhD”

THE MCKEOWN THESIS STATES that the enormous increase in population and dramatic improvements in health that humans have experienced over the past 2 centuries owe more to changes in broad economic and social conditions than to specific medical advances or public health initiatives. The thesis gives center stage to social conditions as root causes of the health of populations. On the basis of new data and numerous revisitations, however, Colgrove2 tells us that the thesis has been “overturned” and the theory “discredited.” Whether, then, the idea that social conditions require prominence in any complete understanding of the health of populations? When we turn away from “the thesis,” do we accept an “antithesis” asserting that the role of social conditions is insignificant?

116. All are true about world health report 2008 except?

A. Social reforms

B. Leadership reforms

C. Polices reforms

Online support at www.pgpyrexia.co.cc

Free monthly AIIMS/ALL INDIA mock exams @ www.pgpyrexia.lefora.com.

Visit today!! Page 39

D. Economic reforms

Ans: D. Economic reforms

Ref:WORLD HEALTH REPORT 2008

4sets of PHC reforms:

• Solidarity and social inclusions:Universal coverage reforms • People centred care:Service reforms • Health authorities:Leadership reforms • communities where health is promoted and protected:Public policy reforms

Here social reforms are mentioned under universal coverage reforms.Nowhere does it mention economic reforms.

117. All are non deliberate measures for control of mosquito except?

A. Use of alkaline soap water

B. Use of larvicidal agents

C. Community participation

D. use of bed nets for mosquito

Ans: A. Use of alkaline soap water in factory

[Ref: Park]

118. People were separated into 5 sub groups. People were selected randomly from these sub groups. What type of sampling was done?

A. Simple random sampling

B. Stratified Sampling

C. Cluster sampling

D. Systematic sampling

Ans: B. Stratified Sampling

Stratified sampling is a probability sampling technique wherein the researcher divides the entire population into different subgroups or strata, then randomly selects the final subjects proportionally from the different strata.

119. About Human Development Index, all are true except?

A. Life expectancy at birth

B. Life expectancy at 1 year of age

C. Education

D. GDP

Ans: B. Life expectancy at 1 year of age [REPEAT]

Online support at www.pgpyrexia.co.cc

Free monthly AIIMS/ALL INDIA mock exams @ www.pgpyrexia.lefora.com.

Visit today!! Page 40

Mnemonic for HDI:

“KILL”

K-knowledge (literacy)

I-Income (GDP)

L-longevity

L- Life expectancy at birth

120. Which is not true regarding diet modification recommended in high cardiovascular risk group?

A. Cholesterol less then 100mg / 1000kcal / day

B. Avoid alcohol

C. Fat intake < 10% of total calories

D. Salt limitation to less than 5 gm

Ans: C. Fat intake < 10% of total calories

Ref: American Heart association 2006 guidelines on diet in cardiovascular disease.

These guidelines confirm the options 1/2/4...regarding option 3....it is the saturated fat content which should be less that 10%.the total fat intake should be less than 30% of the overall calorie intake.

ORTHOPEDICS

121. Blount’s disease is:

A. Genu valgum

B. Genu varum

C. Genu recurvatum

D. Menisceal injury

Ans: B. Genu varum

Blount's disease is a specific form genu varum of affecting infants and adolescents

122. First structure to be fixed after amputation is?

A. Bone fixing

B. Arterial repair

C. Venous repair

D. Nerve repair

Ans: A. Bone fixing

“If the part is deemed to be replantable, bone fixation is performed first. Most commonly, fixation with k-wires is performed. The wires span the site where the bone has been cut, to hold the bone in rigid fixation. “

Online support at www.pgpyrexia.co.cc

Free monthly AIIMS/ALL INDIA mock exams @ www.pgpyrexia.lefora.com.

Visit today!! Page 41

123. A teenaged girl complains of pain in knee on climbing stairs and on getting up after sitting for a long time. What is the probable diagnosis?

A. Chondromalacia

B. Plica syndrome

C. Bipartite patella

D. Patello-femoral osteoarthritis

Ans: A. Chondromalacia

Ref: Wheeler’s textbook

Chondromalacia of the Patella

- Chondromalacia describes softening & fissuring of articular hyaline cartilage;

- Chondromalacia may result from an excessive load on patellofemoral joint, but disuse may be a contributing factor;

- Most common in young women

- contributing factors:

- Weakness and tightness of quadriceps muscle;

- Genu valgum;

- Increased Q angle

- Patella Alta

Clinical Features and Exam:

- Pts may report anterior knee pain, esp. while climbing stairs;

- Compression of patella may cause pain along medial & lateral retinacula & patellar ligament;

- Compression of the patella during flexion & extension of knee may elicit crepitation and discomfort;

- Patellar tracking

124. A 6 year old child presents with pain in hip in femoral triangle region. X-ray does not reveal any abnormality. What is the next step?

A. USG

B. MRI

C. Aspiration

D. Traction

Ans: B. MRI

Best suitable DD in this case is Legg-Calve Perthe disease.(since fever and inflammatory symptoms are not given,transient synovitis of hip and septic arthritis can be excluded)

So the next best investigation as we all knoe is MRI

125. The distant metastasis of bone can be best detected by:

Online support at www.pgpyrexia.co.cc

Free monthly AIIMS/ALL INDIA mock exams @ www.pgpyrexia.lefora.com.

Visit today!! Page 42

A. MRI

B. Bone scan

C. CT

D. Intravenous venogram

Ans: B. Bone scan [REPEAT]

126. A 65 yrs old lady presented with a swollen and painful knee. On examination, she was found to have grade III osteoarthritic changes. What is the best course of action?

A. Conservative management

B. Arthroscopic washing

C. Partial knee replacement

D. Total knee replacement

Ans: D. Total knee replacement

The grading of osteoarthritis: (Ahlback)

Grade 1: joint space narrowing (<3mm)

Grade 2: joint space obliteration

Grade 3: minor bone attrition (0-5 mm)

Grade 4: moderate bone attrition (5-10 mm)

5: severe bone attrition (>10 mm)

The best option to pick up is TKR only..

We have look up the patient's symptamatology in this case.

The GOLDEN POINT to remember here is:

"Total-knee replacemnt is the most cost effective approach to overcome the symptamatology of osteoarthritis"

Ref: Brigham and Women's Experts' Approach to Rheumatology By Jonathan S. Coblyn, Bonnie Bermas, M.D., Michael Weinblatt, Simon Helfgott

Online support at www.pgpyrexia.co.cc

Free monthly AIIMS/ALL INDIA mock exams @ www.pgpyrexia.lefora.com.

Visit today!! Page 43

OPHTHALMOLOGY

127. Late onset endophthalmitis after lens implantation is caused by?

A. Staphylococcus epidermidis

B. Pseudomonas

C. Streptococcus pyogenes

D. Propionibacterium acnes

Ans: D. Propionibacterium acnes

Ref: Retina: color atlas and synopsis of clinical ophthalmology By Allen C. Ho

Online support at www.pgpyrexia.co.cc

Free monthly AIIMS/ALL INDIA mock exams @ www.pgpyrexia.lefora.com.

Visit today!! Page 44

128.I n superior oblique palsy diplopia is :

A. Horizontal and down

B. Horizontal and up

C. Vertical and down

D. Vertical and up

Ans: C. Vertical and down

Ref: Neurology in Clinical Practice: Principles of diagnosis and management by Walter George Bradley

In superior oblique palsy the most common diplopia is isolated vertical diplopia especially on looking down…..

Horizontal diplopia may also be found…

129. A 5 yr old boy presented with leukocoria in right eye ball, while other eye had 2-3 small lesions in the periphery. What will be the ideal management for this patient?

A. Enucleation of both eyes

B. Enucleation of right eye & cryotherapy for the other eye

C. Enucleation for right eye and radiotherapy for the other eye

D. 6 cycles of chemotherapy

Ans: B. Enucleation of right eye & cryotherapy for the other eye

Right eye ball dense leucocoria suggest that retinoblastoma has right eye has involved more than half of globe,for which we know Enucleation is the treatment.

In the other eye, only 2-3 peripheral lesions are found.For this cryotherpy is the treatment of choice.

“Cryotherapy is used for the treatment of small equatorial and peripheral lesions, measuring no more than 3.5 mm in base and not >2 mm thickness”

(Ref: Cancer in Children By William L. Carroll, William Larkin Carroll, Jonathan L. Finlay-page 448)

130. A patient presented with sudden onset of floaters and sensation of falling of a curtain in front of the eye. Which one of the following is the appropriate diagnosis?

A. Retinal detachment

B. Eales disease

C. Vitreous haemorrhage

D. Macular burn

Ans: A. Retinal detachment

Ref: Physician Assistant's Clinical Companion By Springhouse

• Initial symptoms commonly include the sensation of a flashing light (photopsia) related to retinal traction and often accompanied by a shower of floaters (RPE) and vision loss.

• Over time, the patient may report a shadow in the peripheral visual field, which, if ignored, may spread rapidly to involve the entire visual field in a matter of days. Vision loss may be filmy, cloudy, irregular, or curtainlike.

Online support at www.pgpyrexia.co.cc

Free monthly AIIMS/ALL INDIA mock exams @ www.pgpyrexia.lefora.com.

Visit today!! Page 45

Important factsheet:

Floaters are a very common visual symptom in the population; thus, distinguishing their etiology requires eliciting a detailed history.

1. The sudden onset of one large floater in the center of the visual axis indicates posterior vitreous detachment (PVD). The patient observes a circular floater when the vitreous detaches from its annular ring surrounding the optic nerve.

2. Numerous curvilinear opacities indicate vitreous degeneration, which is considered a normal aspect of a mature eye. More ominous and concerning is the description of hundreds of tiny black specks appearing before the eye. This is pathognomonic for vitreous hemorrhage, resulting from disruption of a retinal vessel caused by a retinal tear or mechanical traction of a vitreoretinal adhesion.

3. A few hours after the initial shower of black spots, the patient can note cobwebs that result from blood forming irregular clots.

4. Generally, the new onset of floaters associated with flashing lights indicates a retinal tear until proven otherwise.

131. All are seen in Argyll Robertson pupil except?

A. Near reflex normal

B. Direct reflex absent

C. Consensual reflex normal

D. Vision normal

Ans: C. Consensual reflex is normal

Argyll Robertson pupil

This is caused by neurosyphilis and is characterised by the following:

• Involvement is usually bilateral but asymmetrical

• The pupils arc small and irregular.

• Light-near dissociation. • The pupils are very difficult to dilate.

132. Which of the following is the most probable diagnosis in a young patient with loss of central vision and a normal ERG with no family history?

A. Best's disease

B. Stargardt's disease

C. Retinitis pigmentosa

D. Macular hole

Ans: B. Stargardt disease

Clincal ophthalmology by Vaughan and Kanski Opthalmology:

Stargardt’s Disease

The presentation is in the first to second decades with bilateral, gradual impairment of central vision which may be out of proportion to the macular changes. so that thechild may be suspected of malingering.

Online support at www.pgpyrexia.co.cc

Free monthly AIIMS/ALL INDIA mock exams @ www.pgpyrexia.lefora.com.

Visit today!! Page 46

Both Eale’s and Stargardt’s can manifest with loss of central vision and normal ERG....but stargardt is more common and it is autosomal recessive wheras Eales is AD and less common...

ENT:

133. All are true statements regarding use of sodium fluoride in the treatment of otosclerosis except?

A. It inhibits osteblastic activity

B. Used in active phase of otosclerosis when schwartz

C. Nephritis is a contraindication

D. It decreases the release of osteolytic enzymes

Ans: A. Inhibits osteblastic activity [REPEAT]

134. A patient had running nose and pain over medical aspect of eye. He later developed chemosis,protosis and diplopia on abduction of right eye with congestion of optic disc. What is the probable diagnosis?

A. Acute ethmoidal sinusitis

B. Orbital cellulitis

C. Cavernous sinus thrombosis.

D. Orbital apex syndrome

Ans: C. Cavernous sinus thrombosis [REPEAT]

135. Endolymphatic Hydrops is seen in

A. Meniere’s disease

B. Otosclerosis

C. Acoustic neuroma

D. Sinusitis

Ans: A. Meniere’s disease

136. A fire breaks out during laser vocal cord surgery. What is not to be done?

A. Pouring sterile water

B. Removing endotracheal tube

C. 100% oxygen after discontinuing anesthetic gases

D. Treatment with steroid & antibiotic

Ans: C. 100% oxygen after discontinuing anesthetic gases

Ref: Endolaryngeal surgery By Bruce Benjamin

Online support at www.pgpyrexia.co.cc

Free monthly AIIMS/ALL INDIA mock exams @ www.pgpyrexia.lefora.com.

Visit today!! Page 47

137. onodi cells & haller cells are associated with the following structures respectively?

A. Optic nerve & orbital floor

B. Optic nerve & interal carotid artery

C. Internal carotid artery& optic nerve

D. Orbital floor & internal carotid artery

Ans: A. Optic nerve & orbital floor

Onodi cell(sphenoethomoid cell):

This is formed by lateral and posterior pneumatization of the most posterior ethmoid cells over the sphenoid sinus.

Because the Onodi cells are posterior ethmoid cells that are positioned superolateral to the sphenoid sinus ,the optic nerve & carotid artery may often course through the lateral aspect of onodi cell instead of sphenoid sinus proper.

Haller cell (infraorbital cell or infraorbital extension of ethmoid cell):

The Haller cell is usually situated below the orbit in the roof of the maxillary sinus.(i.e., in the orbital floor).

It is a pneumatized ethmoid cell that projects along the medial roof of the maxillary sinus.

As it is closely related to ethmoidal infundibulum enlarged Haller cells may contribute to narrowing of the ethmoidal infundibulum and recurrent sinus disease.

PAEDIATRICS:

138. . Baby born at 33 weeks / 1.5 kg should be started on?

A. Nil oral and IV fluids

B. Oral nasogastric tube/alternate oral route

C. IV fluids and oral feeding

D. TPN

Ans: B. Oral nasogastric tube/alternate oral route

Online support at www.pgpyrexia.co.cc

Free monthly AIIMS/ALL INDIA mock exams @ www.pgpyrexia.lefora.com.

Visit today!! Page 48

Ref: IAP paediatrics

Birth weight

(g)

Infant sucking response

Good suck Slow suck No suck

<1600 NGF. Try BF as soon as possible NGF. Try BF as soon as possible NGF. Try BF as soon as possible

1600-2000

Breast feed Breast Feed

+ cup/spoon

NGF. Try BF as soon as possible

Ad lib feeding is recommended in older well babies

NGF-Nasogastric feed,BF-breast feed

139. A 18 year old male presents with hemetemesis, melena and splenomegaly. What is the probable initial diagnosis?

A. NCPF

B. Cirrhosis

C. Malaria with DIC

D. Extra hepatic portal venous obstruction

Ans: A. NCPF [REPEAT]

140. Earliest symptom of GERD in an infant is?

A. Respiratory distress

B. Upper GI bleed

C. Regurgitation

D. Obstruction

Ans: C. Regurgitation

Ref: Nelson

Infantile reflux manifests more often with regurgitation (especially postprandially), signs of esophagitis (irritability, arching, choking, gagging, feeding aversion), and resulting failure to thrive; symptoms resolve spontaneously in the majority by 12–24 months

141. A child is brought by mother with the complaints of abdominal pain. Physical examination and radiological investigations were normal. There were no signs of intestinal obstruction. The pain was associated with passage of stools.what is the probable diagnosis?

A. Rectal Polyp

Online support at www.pgpyrexia.co.cc

Free monthly AIIMS/ALL INDIA mock exams @ www.pgpyrexia.lefora.com.

Visit today!! Page 49

B. Intussusception

C. Meckel's diverticulum

D. Necrotizing Enterocolitis

Ans: C. Meckel's diverticulum

Intussusception will present with signs of bowel obstruction (ruled out since no radiological and clinical sign in the given case)

Necrotizing Enterocolitis will have sepsis like physical signs and radiographically can also detect it.

Rectal polyp (can be diagnosed on PR examination and radiographically), will present with bleeding per rectum or pain. (Ruled out since Physical examination and radiological investigations were normal)

Now,

“Painless rectal bleeding suggests a Meckel diverticulum; while vomiting, bloody stools, and intermittent crampy abdominal pain suggest intussusception.”

(Ref: Harwood-Nuss' Clinical Practice of Emergency Medicine By Allan B Wolfson)

So In our case Meckel’s diverticulum is the best diagnosis since it has no radiological/physical signs and the fact that it presents with rectal bleed

142. Most important prognostic factor in congenital diaphragmatic hernia?

A. Pulmonary hypertension

B. Timing of surgery

C. Size of defect

D. Gestational Age

Ans: A Pulmonary hypertension

Ref: Nelson

Relative predictors of a poor prognosis include an associated major anomaly, symptoms before 24 hr of age, severe pulmonary hypoplasia, herniation to the contralateral lung, and the need for ECMO.

Serious sequelae include pulmonary function changes, neurodevelopmental delays, and growth retardation.

Pulmonary problems continue to be a source of morbidity for long-term survivors of CDH.

143. A 3.8 kg baby of a diabetic mother developed seizures 16 hours after birth. Most probable cause is?

A. Hypoglycemia

B. Hypocalcemia

C. Birth asphyxia

D. Intra ventricular hemorrhage

Ans: B. Hypocalcemia

This is a very important factbox.Save the entire factbox into your memory:

Online support at www.pgpyrexia.co.cc

Free monthly AIIMS/ALL INDIA mock exams @ www.pgpyrexia.lefora.com.

Visit today!! Page 50

Age of onset of convulsion

First day: Hypoxic-ischemic encephalopathy, cerebral contusion, first day hypocalcaemia, pyridoxine dependency accidental injections of local anesthesia

Between 1-3 days: intracranial hemorrhage, hypoglycemia, narcotic withdrawal, inborn error of metabolism.

Fourth to seventh day : tetany(late onset hypocalcemia), meningitis, torch infection developmental malformation,kernicterus.

144. A child presented with mild fever little breathlessness was treated and she improved over 4 days but later deteriorated again with fever and more breathlessness. x ray showed hyperlucency. diagnosis?

A. Bronchiolitis obliterans

B. Alveolar proteinosois

C. Bronchitis

D. Asthma

Ans: A. Bronchiolitis obliterans [REPEAT]

145. Commonest cause for bilateral proptosis in children?

A. Cavernous haemangioma

B. Rhabdomyosarcoma.?

C. ALL

D. AML

Ans: D. AML

The most common causes of Bilateral ptosis in children is granulocytic sarcoma in AML/MDS and metastatic neuroblastoma

Most common cause of malignant proptosis in children:

• In india—Retinoblastoma • Western countries-Rhabdomyosarcoma

146. A 6 week old male infant was brought in a state of dehydration and shock. Examination revealed hyper pigmentation over the body with normal external genitalia. Blood tests revealed hypoglycemia, Na - 124 mEq/L and K - 7 mEq/L. What is the probable diagnosis?

A. Congenital adrenal hyperplasia

B. Adrenal haemorrhage and shock

C. Acute gastroenteritis with dehydration

D. Pseudohermaphroditism

Ans: A. Congenital adrenal hyperplasia [REPEAT]

Online support at www.pgpyrexia.co.cc

Free monthly AIIMS/ALL INDIA mock exams @ www.pgpyrexia.lefora.com.

Visit today!! Page 51

147. Pentalogy of fallot has which one of following extra entities:

A. ASD

B. VSD

C. RVH

D. Pulmonary stenosis

Ans: A. ASD [REPEAT]

BIOCHEMISTRY:

148. Vitamin K is involved in the posttranslational modification of?

A. Glutamate

B. Aspartate

C. Histidine

D. Tryptophan

Ans: A. Glutamate

Ref: Harper

Vitamin K is the cofactor for the carboxylation of glutamate residues in the post-synthetic modification of proteins to form the unusual amino acid gamma-carboxyglutamate

149. Urea cycle occurs in:

A. Liver

B. G.I.T.

C. Spleen

D. Kidney

Online support at www.pgpyrexia.co.cc

Free monthly AIIMS/ALL INDIA mock exams @ www.pgpyrexia.lefora.com.

Visit today!! Page 52

Ans: A. Liver

Ref: Harper

Urea biosynthesis occurs in four stages: (1) transamination, (2) oxidative deamination of glutamate, (3) ammonia transport, and (4) reactions of the urea cycle . The use of complementary DNA probes has shown that the expression in liver of the RNAs for all the enzymes of the urea cycle increases several fold in starvation.

150. Thiamine deficiency causes decreased energy production because?

A. It is required for the process of tramination

B. It is a co-factor in oxidative reduction

C. It is a co-enzyme for ketolase in pentose phosphate pathway

D. It is a co-enzyme for pyruvate dehydrogenase & alpha ketoglutarate dehydrogenase.

Ans: D. It is a co-enzyme for pyruvate dehydrogenase & alpha ketoglutarate dehydrogenase.

Ref: Harper

Thiamin has a central role in energy-yielding metabolism, and especially the metabolism of carbohydrates . Thiamin diphosphate is the coenzyme for three multi-enzyme complexes that catalyze oxidative decarboxylation reactions: pyruvate dehydrogenase in carbohydrate metabolism; alpha ketoglutarate dehydrogenase in the citric acid cycle; and the branched-chain keto-acid dehydrogenase involved in the metabolism of leucine, isoleucine, and valine.

151. Best test for assessing HCG action?

A. Radioimmunoassay.

B. ELISA

C. Latex test

D. Bioassay

Online support at www.pgpyrexia.co.cc

Free monthly AIIMS/ALL INDIA mock exams @ www.pgpyrexia.lefora.com.

Visit today!! Page 53

Ans: A. Radioimmunoassay

The molecular weight of hCG-α is 18,000, and that of hCG-β is 28,000. hCG is primarily luteinizing and luteotropic and has little FSH activity. It can be measured by radioimmunoassay and detected in the blood as early as 6 days after conception.

152. Which among the following is a cardioprotective fatty acid?

A. Palmitic acid

B. Stearic acid

C. Oleic acid

D. Omega-3 fatty acids

Ans: D. Omega-3 fatty acids

Ref: Harrison

N-3 polyunsaturated fatty acids (n-3 PUFAs) are present in high concentration in fish and in flax seeds. The most widely used n-3 PUFAs for the treatment of hyperlipidemias are the two active molecules in fish oil: eicosapentanoic acid (EPA) and decohexanoic acid (DHA). A lower dose of omega 3 (about 1 g) has been associated with reduction in cardiovascular events in CHD patients and is used by some clinicians for this purpose.

153. What is false about lipoproteins?

A. HDL inhibit oxidation of LDL

B. Oxidized LDL is not present in foam cells

C. HDL is a better predictor of CHD

D. Not recalled

Ans: B.Oxidized LDL is not present in foam cells

Ref: Harper

What are foam cells?

“. When the macrophages become overloaded with oxidized LDL, they become the "foam cells" that are seen in early atherosclerotic lesions. [Ref: Gannong]”

Option A: The export pathway from macrophage foam cells to peripheral cells such as hepatocytes explains part of HDL's antiatherogenic action. (Anti-inflammatory and antioxidant properties may also contribute to HDL's atheroprotective effects.)

Option C: There is also an inverse relationship between HDL (HDL2) concentrations and coronary heart disease, making the LDL:HDL cholesterol ratio a good predictive parameter

154. Transfer of an amino group from an amino acid to an alpha keto acid is done by?

A. Transaminases

B. Aminases

C. Triketoses

D. Transferase

Online support at www.pgpyrexia.co.cc

Free monthly AIIMS/ALL INDIA mock exams @ www.pgpyrexia.lefora.com.

Visit today!! Page 54

Ans: A. Transaminases

155. What factor is responsible for deciding whether an antibody will remain membrane bound or get secreted?

A. RNA splicing

B. Class switching

C. Differential RNA processing

D. Allelic exclusion

Ans: C. Differential RNA processing

Ref: Harrison

The, eukaryotic cells employ alternative RNA processing to control gene expression. This can result when alternative promoters, intron-exon splice sites, or polyadenylation sites are used. Occasionally, heterogeneity within a cell results, but more commonly the same primary transcript is processed differently in different tissues. Alternative splicing and processing results in the formation of seven unique tropomyosin mRNAs in seven different tissues.

156. Not a predisposing factor for atherosclerotic plaque formation?

A. Apo E

B. Alpha 2-macroglobulin

C. Oxidised LDL

D. Increased homocystiene

Ans: B. Alpha 2-macroglobulin

Ref: Robbins

LDL (apo B-100, apo E) receptors occur on the cell surface in pits that are coated on the cytosolic side of the cell membrane with a protein called clathrin. LDL receptor activity on the cell surface is regulated by the cholesterol requirement for membranes, steroid hormones, or bile acid synthesis

Hyperhomocystinemia: Clinical and epidemiologic studies show a strong relationship between total serum homocysteine levels and coronary artery disease, peripheral vascular disease, stroke, and venous thrombosis

157. Gene for expression of protein,yielding max production of enzyme.is ensured by introduction of following gene by virus ?

A. Promoter gene

B. Initiator signal

C. Translation and transcription termination signal

D. Genes for protease inhibitors

Online support at www.pgpyrexia.co.cc

Free monthly AIIMS/ALL INDIA mock exams @ www.pgpyrexia.lefora.com.

Visit today!! Page 55

Ans: A. Promoter gene

Ref: Harper

“It is the association of RNAP with promoters that ensures accurate initiation of transcription”

Since Transcription is the initial process of synthesis of protein production,the best possible answer becomes Promoter gene introduction

158. Basal metabolic rate is closely associated with?

A. Lean body mass .

B. Body surface area

C. Body mass index

D. Sex

Ans: A. Lean body mass [REPEAT]

159. Two plants are grown. One in a fluorescent pigment containing media. Other in fire fly luciferase containing media. Which plant will glow in the dark?

A. Both plants will glow

B. Neither will glow

C. First one will glow

D. Second one will glow

Ans: A. Both plants will glow

Please read blow:

The “glow-in-the-dark” genes come from sources such as fireflies (which contain luciferase) . The gene is attached to a "vector" which should be able to slip into the plant cell and get carried along as a part of the cell's DNA.

The genes taken from the firefly and bacteria are the luciferase enzymes. When these enzymes break down the substrate luciferin, light is produced. These genes need a blue light (or UV light) and calcium in order to glow in the dark.

Many references also suggest that Green fluorescent pigment also has the ability of “glow-in-dark” and the so the plant will glow in the dark.

160. Which among the following is not a cause of fasting hypoglycemia?

A. Glucagon excess

B. Glucose 6 phospatase deficiency

C. Ureamia

D. Glycogen synthase deficiency

Ans: A. Glucagon excess

Ref: Harrison Glucose 6 phosphatase defeciency leads to inability to mobilize glucose and hence can cause

Glycogen synthase def leads to glycogen deficiency and hence during fasting patient goes hypo

Online support at www.pgpyrexia.co.cc

Free monthly AIIMS/ALL INDIA mock exams @ www.pgpyrexia.lefora.com.

Visit today!! Page 56

In uremia liver metabolism is hampered and hence hypo is possible

In glucagon excess there is HYPERglycemia and hence the answer.

SURGERY:

161. A 55 year old man presents with history of 5 episodes of hematuria each lasting for about 4-5 days in the past 5 years. What will be the best investigation to arrive at a diagnosis?

A. Urine examination and microscopy

B. X-ray KUB

C. Abdominal USG

D. DTPA scan

Ans: A. Urine examination and microscopy

Ref: Harrison

162. A patient has a single kidney with an exophytic mass of 4 cm size at it’s lower pole. Which among the following is the best course of action?

A. Partial nephrectomy.

Online support at www.pgpyrexia.co.cc

Free monthly AIIMS/ALL INDIA mock exams @ www.pgpyrexia.lefora.com.

Visit today!! Page 57

B. Radical nephrectomy with dialysis

C. Radical nephrectomy with immediate renal transplant

D. Observation

Ans: A. Partial nephrectomy

Ref: Schwartz

“Patients with localized disease may be cured with either partial or radical nephrectomy . The oncologic efficacy of partial nephrectomy (nephron sparing) appears to be similar to that of radical nephrectomy. However, those patients with larger tumors or with a more central tumor location may be at increased risk for surgical complications. Nephron-sparing surgery should be considered in all patients, if feasible, as those patients undergoing a radical nephrectomy are at risk for future chronic kidney disease

163. A patient with head injury on examination revealed eye opening in response to pain, inappropriate words and pain localisation. Calculate GCS?

a. 10

b. 8

c. 12

d. 14

Ans: A. 10

Revise this table n calculate.[Ref:Harrison]

Glasgow Coma Scale for Head Injury

Eye opening (E) Verbal response (V)

Spontaneous 4 Oriented 5

To loud voice 3 Confused, disoriented 4

To pain 2 Inappropriate words 3

Nil 1 Incomprehensible sounds 2

Best motor response (M)

Nil 1

Obeys 6

Localizes 5

Withdraws (flexion) 4

Abnormal flexion posturing 3

Extension posturing 2

Nil 1

164. . A surgeon removed the part of liver to the left of the falciform ligament. Which segments have been removed?

A. 1 & 4a

B. 2 & 3.

Online support at www.pgpyrexia.co.cc

Free monthly AIIMS/ALL INDIA mock exams @ www.pgpyrexia.lefora.com.

Visit today!! Page 58

C. 1 & 4b

D. 5 & 8

Ans: B. 2 & 3.

165. A patient with stab injury to abdomen with omentum protrusion in umbilical area ,vitals stable.immediate next step ?

A. FAST

B. Laparotomy

C. Suturing with wound exploration

D. CECT abdomen

Ans: C. Suturing with wound exploration

Ref: Sabiston

“Approximately 25% of all trauma victims will require abdominal exploration”

Stab wounds to the abdomen, however, carry a significantly lower risk of intra-abdominal organ injury than do gunshot wounds, and several studies have recently favored a more selective approach, as opposed to mandatory exploratory laparotomy.

“The impetus for nonoperative management of solid organ injury in stable blunt trauma patients has expanded to penetrating trauma as well.”

VERY IMPORTANT:

Hypotensive patients with isolated penetrating abdominal trauma who are hypotensive or in shock or have peritoneal signs should go to the operating room despite the mechanism of injury. Stab wound victims without peritoneal signs, evisceration, or hypotension benefit from wound exploration and DPL. Gunshot wound victims should generally undergo exploration

***So, as the question states no signs of hypotension or evisceration, we can go with the local wound exploration as the answer.

166. A lady who presented with hematuria on evaluation was found to have stage 2 transitional cell carcinoma of bladder. Which of the following is true?

A. 70% chance of requiring cystectomy in next 5 yrs.

Online support at www.pgpyrexia.co.cc

Free monthly AIIMS/ALL INDIA mock exams @ www.pgpyrexia.lefora.com.

Visit today!! Page 59

B. Cystoscopic fulguration will have to be done

C. A 10 year history of smoking is not a risk factor

D. There is no chemotherapy available

Ans: A. 70% chance of requiring cystectomy in next 5 yrs.

Ref: Bailey and Sabiston

Option.C: Cigarette smoking is associated with a two to three-fold excess risk, and certain genetic polymorphisms for N-acetyl tranferase, glutathione transferase and one of the cytochrome P45Os (CYP2D6) may increase the risk of occupationally acquired bladder cancer.

Option A&D: The use of systemic chemotherapy by means of a combination of agents using cis-platinum, methotrexate, adriamycin and vinblastine (MVAC) in addition to conventional treatment is presently being studied.

Radical cystectomy and pelvic lymphadenectomy.: This is now standard treatment for localised pT2—pT3 disease without evidence of secondary spread or of cis which has not responded to BCG.[Ref: Bailey]

Option B: Fulguration can be used only for lesions below the peritoneal reflection. Obviously, this technique cannot provide a specimen to assess the pathologic stage because the tumor and margins are disintegrated by fulguration. The procedure is reserved for patients with a prohibitive operative risk and limited life expectancy.

167. Gold standard test for diagnosis of laryngopharyngeal reflux?

A. 24 hr double probe pH monitoring

B. Flexible endoscopy

C. Barium swallow

D. Laryngoscopy

Ans: A. 24 hr double probe pH monitoring

Ref: Sabiston

There are two mechanisms that have been proposed as the cause of reflux-induced respiratory symptoms. The reflux theory suggests that these symptoms are the direct result of laryngopharyngeal exposure and aspiration of gastric contents. The reflex theory suggests that the vagal-mediated afferent fibers result in bronchoconstriction during episodes of distal esophageal acidification.

The gold standard for diagnosing and quantifying acid reflux is the 24-hour pH test

168. During TURP, surgeon takes care to dissect above the verumontenum so as to prevent injury to?

A. External urethral sphincter

B. Urethral crest

C. Prostatic utricle

D. Yrigone of bladder

Ans: A. External urethral sphincter

Ref: Bailey

Coagulation of bleeding points can be accurately achieved and damage to the external sphincter is avoided provided one uses the verumontanum as a guide to the most distal point of the resection.

Online support at www.pgpyrexia.co.cc

Free monthly AIIMS/ALL INDIA mock exams @ www.pgpyrexia.lefora.com.

Visit today!! Page 60

169. Most common site of obstruction after TURP?

A. Navicullar fossa

B. Bulb

C. Prostatic membranous urethra

D. Bladder neck

Ans: D. Bladder neck

Ref: Bailey

Bladder neck contracture: Occasionally a dense fibrotic stenosis of the bladder neck occurs following overaggressive resection of a small prostate. It may be due to the overuse of the coagulating diathermy. Transurethral incision of the scar tissue is necessary.

Bladder neck contractures and strictures are the most common reasons for continued obstruction after TURP. Contractures have especially been a problem after resection of small prostates

170. Which is the most reliable objective sign of identifying pulmonary plethora in chest X-ray?

A. Diameter of the main pulmonary artery >16mm

B. Diameter of the lt pulmonary artery >16mm

C. Diameter of the descending Rt pulmonary artery >16mm

D. Diameter of the descending Lt pulmonary artery >16mm

Ans: C. Diameter of the descending Rt pulmonary artery >16mm

171. Which organ obtained from a cadaver is not used for transplantation?

A. Blood vessel

B. Lung

C. Liver

D. Bladder

Ans: D. Bladder

Online support at www.pgpyrexia.co.cc

Free monthly AIIMS/ALL INDIA mock exams @ www.pgpyrexia.lefora.com.

Visit today!! Page 61

Organs that can be transplanted are the heart, kidneys, liver, lungs, pancreas, intestine, and thymus. Tissues include bones, tendons (both referred to as musculoskeletal grafts), cornea, skin, heart valves, and veins.

172. A 50 yr lady has history of sprained ankle 2 months back followed by recovery. She now complains of severe pain in that ankle with inability to flex that foot. Physician notes edema and shiny skin in local examination. What is the probable diagnosis:

A. Fibromyalgia

B. Complex regional pain syndrome 1

C. Complex regional pain syndrome 2

D. Peripheral neuropathy

Ans: B. Complex regional pain syndrome 1

The most widely accepted criteria for the diagnosis were published by the International Association for the Study of Pain (IASP). It lists the diagnostic criteria for complex regional pain syndrome I (CRPS I) (RSDS) as follows:

• 1.The presence of an initiating noxious event or a cause of immobilization • 2.Continuing pain, allodynia (perception of pain from a nonpainful stimulus), or hyperalgesia disproportionate

to the inciting event • 3.Evidence at some time of edema, changes in skin blood flow, or abnormal sudomotor activity in the area of

pain • 4.The diagnosis is excluded by the existence of any condition that would otherwise account for the degree of

pain and dysfunction.

According to the IASP, CRPS II (also known as causalgia) is diagnosed as follows:

• 1.The presence of continuing pain, allodynia, or hyperalgesia after a nerve injury, not necessarily limited to the distribution of the injured nerve

• 2.Evidence at some time of edema, changes in skin blood flow, or abnormal sudomotor activity in the region of pain

• 3. The diagnosis is excluded by the existence of any condition that would otherwise account for the degree of pain and dysfunction.

Note that the primary difference between type I and type II is the identification of a definable nerve injury

173. Patient presents with epigastric pain which radiates to the back and relieved by food, patient have history of such pain in past for which he was taking analgesics and in past 5 years 2 times operated for duodenal ulcer. What is the diagnosis

A. Gastric ulcer

B. Duodenal ulcer

C. Chronic pancreatitis

D. Not recalled

Ans: B. Duodenal ulcer

Ref: Sabiston

The most common symptom associated with duodenal ulcer disease is midepigastric abdominal pain that is usually well localized. The pain is usually tolerable and frequently relieved by food. When the pain becomes constant, this suggests that there is deeper penetration of the ulcer, and referral of pain to the back is usually a sign of penetration into the pancreas.

Online support at www.pgpyrexia.co.cc

Free monthly AIIMS/ALL INDIA mock exams @ www.pgpyrexia.lefora.com.

Visit today!! Page 62

174. Patient having pain in epigastrium which radiates to back, serum amylase is normal, on x ray gall stone seen and pancrease appeares bulky..

A. Acute Pancreatitis

B. Acute cholecyctitis

C. Duodenal ulcer

D. Gastric ulcer

Ans: A. Acute Pancreatitis

Ref: Schwartz

All episodes of acute pancreatitis begin with severe pain, generally following a substantial meal. The pain is usually epigastric, but can occur anywhere in the abdomen or lower chest. It has been described as "knifing" or "boring through" to the back, and may be relieved by the patient leaning forward.

Serum amylase concentration increases almost immediately with the onset of disease and peaks within several hours. It remains elevated for 3 to 5 days before returning to normal

175. A patient presents with fever and abdominal pain. Clinical examination reveals hepatomegaly extending 4 finger breadths below the costal margin. USG reveals a 5cm*5cm*4cm hypodense lesion 1cm deep to liver surface. Tests for hydatid disease were -ve. Best course of action is?

A. Multiple aspirations, anti-amoebics and antibiotics

B. Catheter drainage with anti-amoebics and antibiotics

C .Hepatectomy

d. Medical management with anti-amoebics and antibiotics

Ans: A. Multiple aspirations, anti-amoebics and antibiotics

Ref: Sabiston

The dilemma is to say whether this is a pyogenic or an amoebic liver abscess.

The statement that “solitary”abscess makes us think it is amoebic.

But let us remember that:

“50%pyogenic and 80% amebic abcess are solitary”

So we are not sure about the diagnosis (also remember that serology –ve for hydatid does not rule out hydatid)

If we are uncertain then the TOC should encompass both ---drain and give antiamebics

i.e., drainage +antibiotics in suspicion of pyogenic abscess AND antiamoebics in suspicion of amoebic abscess

(NOTE: mainstay of treatment for amebic abscesses is metronidazole (750 mg orally three times per day for 10 days), which is curative in more than 90% of patients***-VERY IMPORTANT

176. A 5 year old child presented with ballooning of perpuce while micturition. Perpuce adhesions were present. What is the best treatment for him?

A. Adhesiolysis and dilatation

Online support at www.pgpyrexia.co.cc

Free monthly AIIMS/ALL INDIA mock exams @ www.pgpyrexia.lefora.com.

Visit today!! Page 63

B. Circumcision

C. Dorsal slit

D. Conservative

Ans: B. Circumscision [REPEAT]

177. True regarding ranula?

A. It is also known as epulis

B. It is a cystic swelling in the floor of mouth

C. It is a type of thyroglossal cyst

D. It is a type of mucus retention cyst

Ans: B&D. It is a cystic swelling in the floor of mouth & . It is a type of mucus retention cyst

Ref: Robbins

A ranula is histologically identical to a mucocele. However, this term is reserved for mucoceles that arise when the duct of the sublingual gland has been damaged. A ranula can become extremely large and develop into a “plunging ranula” when it dissects its way through the connective tissue stroma connecting the two bellies of the mylohyoid muscle

178. Time of occurrence of secondary hemorrhage after tonsillectomy?

A. 24 hrs

B. 6 days

C. 12 days

D. 12 hrs

Ans: B. 6 days

Ref: CMDT 2011

Secondary bleeding 5–8 days postoperatively is far more common than bleeding in the first 24 hours

179. Which of the following is a contraindication for medical treatment in gallstones?

A. Radio opaque stones.

B. Radiolucent stones

C. Normal functioning gall bladder

D. Small stones

Ans: A. Radio opaque stones

Ref: Harrison

“In carefully selected patients with a functioning gallbladder and with radiolucent stones <10 mm in diameter, complete dissolution can be achieved in ~50% of patients within 6 months to 2 years with UDCA at a dose of 8–10 mg/kg per day”

Online support at www.pgpyrexia.co.cc

Free monthly AIIMS/ALL INDIA mock exams @ www.pgpyrexia.lefora.com.

Visit today!! Page 64

180. Triple phosphate stone seen in infection with?

A. Proteus

B.E.coli

C.Klebsiella

D.Staphylococcus

Ans: A. Proteus [REPEAT]

181. A child with suspected aspiration of food particles into larynx.first step in management?

A. Cricothyroidectomy

B. Emergency tracheostomy

C. Humidified oxygen

D. Heimlich maneuvere

Ans: D. Heimlich maneuvere

Ref: Nelson

“Complete obstruction rapidly asphyxiates the child unless promptly relieved with the Heimlich maneuver”

• Use of the Heimlich maneuver has improved the mortality rate of patients with complete airway obstruction, but use of it in patients with partial obstruction may produce complete obstruction.

182. Ideal age for surgery in unilateral undescended testis is?

A. 6 months

B. 12 months

C. 24 months

D. 36 months

Ans: A. 6 months

Though are equally good no. of evidences for both 6 months and 12 months,many sources including IAP recommend earlier age of orchidopexy.

So, it is better to mark 6 months if repeated.

183. Multiple sebaceous adenomas seen in:

A. Gardner’s syndrome

B. Turcot syndrome

C. Muir-Torre syndrome

D. Cowden syndrome

Ans: C. Muir-Torre syndrome

Online support at www.pgpyrexia.co.cc

Free monthly AIIMS/ALL INDIA mock exams @ www.pgpyrexia.lefora.com.

Visit today!! Page 65

Ref: Sabiston

HEREDITARY NONPOLYPOSIS COLON CANCES

Other Clinical Features: Muir-Torre variant: Sebaceous adenomas, keratoacanthomas, sebaceous epitheliomas, and basal cell epitheliomas

189. Lines of blaschko are:

A. Lymphatics

B. Blood vessel

C. Nerves

D. Lines of development.

Ans: D. Lines of development

Reference: Avery's neonatology:

“During development, skin and its structures are thought to migrate along the "lines of Blaschko" . Most authorities believe that Blaschko lines are an expression of epidermal and not dermal migration”

190. Superior vena caval syndrome is most commonly caused by?

A. Lymphoma

B. Small cell lung ca.

C. Non small cell lung ca

D. Secondary tumours

Ans: B. Small cell lung ca.

Ref: Harrisons

Oncological emergencies

“Lung cancer, particularly of small cell and squamous cell histologies, accounts for approximately 85% of all cases of malignant origin.”

PATHOLOGY:

191. Necrotizing lymphadenitis is seen in?

A. Kimura disease

B. Kikuchi disease

C. Hodgkin disease

D. Castelman disease

Ans: B. Kikuchi disease

Ref: Robbins

Online support at www.pgpyrexia.co.cc

Free monthly AIIMS/ALL INDIA mock exams @ www.pgpyrexia.lefora.com.

Visit today!! Page 66

Among HIV-infected patients, granulomas usually are not seen. Differential diagnosis includes a variety of infectious conditions, neoplastic diseases such as lymphomas or metastatic carcinomas, and rare disorders like Kikuchi disease (necrotizing histiocytic lymphadenitis), Kimura's disease, and Castleman's disease

192. NARP syndrome is an example of?

A. Mitochondrial function disorder

B. Glycogen storage disorder

C. Lysosomal storage disorder

D. Lipid storage disorder

Ans: A. Mitochondrial function disorder [REPEAT]

NARP= neuropathy, ataxia, and retinitis pigmentosa

193. Sterile pyuria is present in?

A. Renal tuberculosis

B. Chronic hydronephrosis

C. Wilm's tumour

D. Neuroblastoma

Ans: A. Renal tuberculosis

Ref: Robbins

Pyuria in the absence of bacteriuria (sterile pyuria) may indicate infection with unusual agents such as C. trachomatis, U. urealyticum, or Mycobacterium tuberculosis or with fungi.

194. True about platelet function defect?

A. Normal platelet count with prolonged bleeding time

B. Thrombocytosis with prolonged bleeding time

C. Both platetelet count and bleeding time is decreased

D. Normal platelet count and bleeding time

Ans: A. Normal platelet count with prolonged bleeding time

Ref: Robbins

“Essentially in platelet function disorders,the total no. of platelets is not changed,RATHER there there is a defective function manifested by prolonged bleeding time”

195. Poor prognostic factor for ALL is?

A. Hyperdiploidy

B. t(9;22) t(4;11)

C. 2-8 yrs of age

Online support at www.pgpyrexia.co.cc

Free monthly AIIMS/ALL INDIA mock exams @ www.pgpyrexia.lefora.com.

Visit today!! Page 67

D. TLC < 50000

Ans: B. t (9;22) t(4;11)

Ref:Robbins

Several factors have been consistently associated with a worse prognosis:

(1) age under 2, largely because of the strong association of infantile ALL with translocations involving the MLL gene

(2) presentation in adolescence or adulthood

(3) peripheral blood blast counts greater than 100,000, which probably reflects a high tumor burden

(4) the presence of particular cytogenetic aberrations such as the t(9;22) (the Philadelphia chromosome).

196. Most potent activator of T cells?

A. B cells

B. Follicular dendritic cells

C. Mature dendritic cells

D. Macrophages

Ans: C. Mature dendritic cells [REPEAT]

197. Psammoma bodies are seen in all except?

A. Follicular carcinoma thyroid

B. Papillary carcinoma thyroid

C. Cystadenocarcinoma

D. Meningioma

Ans: A. Follicular carcinoma thyroid [REPEAT]

198. Difference between follicular carcinoma and follicular adenoma is?

A. Vascular invasion

B. Histological appearance

C. FNAC

D. Clinical features

Ans: A. Vascular invasion

Ref: Robbins

“Nuclear features are helpful in distinguishing papillary from follicular neoplasms, they are of little value in distinguishing follicular adenomas from minimally invasive follicular carcinomas. This distinction requires extensive histologic sampling of the tumor-capsule-thyroid interface to exclude capsular and/or vascular invasion”

199. Which is not an autoimmune disease?

Online support at www.pgpyrexia.co.cc

Free monthly AIIMS/ALL INDIA mock exams @ www.pgpyrexia.lefora.com.

Visit today!! Page 68

A. SLE

B. Grave's disease

C. Myasthenia gravis

D. Sickle cell disease

Ans: D. Sickle cell disease

Sickle cell disease: It is a Red cell disorder assosciated with structurally abnormal globins (hemoglobinopathies)

200. Not a disorder of protein misfolding?

A. Alzheimer's disease

B. Tuberculosis

C. Cystic fibrosis

D. CJD

Ans: B. Tuberculosis

Examples of Protein misfolding diseases

(Hot topic for AIIMS may 2011)

1. Prion diseases including CJD,Kuru,Scrapie,FFI,etc.. 2. cystic fibrosis (misfolded CFTR protein), 3. Marfan syndrome (misfolded fibrillin), 4. Fabry disease (misfolded alpha galactosidase), 5. Gaucher’s disease (misfolded beta glucocerebrosidase) 6. retinitis pigmentosa 3 (misfolded rhodopsin) 7. von Hippel Lindau protein 8. Alzheimer’s disease (deposits of amyloid beta and tau), 9. Type II diabetes (depositis of amylin), 10. Parkinson’s disease(deposits of alpha synuclein)

201. A young male presented with history of fever and a nodule in the leg. Histopathology of the nodule revealed foamy histiocytes and neutrophillic infiltrate in the dermis. Most probable diagnosis is?

A. Sweet's syndrome

B. Rosai Dorfman disease

C. Erythema Nodosum Leprosum

D. Erythema nodosum

Ans: A. Sweet's syndrome [REPEAT]

202. Erythema nodosum is seen in all except:

A. Pregnancy

B. Tuberculosis

C. SLE

Online support at www.pgpyrexia.co.cc

Free monthly AIIMS/ALL INDIA mock exams @ www.pgpyrexia.lefora.com.

Visit today!! Page 69

D. Chronic pancreatitis.

Ans: D. Chronic pancreatitis.

Ref: Emedicine

Currently, the most common cause of erythema nodosum is streptococcal infection in children and streptococcal infection and sarcoidosis in adults.

Other causes are:

•Bacterial infections: Streptococcal infections are one of the most common causes of erythema nodosum. Tuberculosis was an important cause in the past, but it has decreased dramatically as a cause for erythema nodosum

•Fungal infections: Coccidioidomycosis (San Joaquin Valley fever) is the most common cause of erythema nodosum in the American Southwest

•Drugs: Sulfonamides and halide agents are an important cause of erythema nodosum. Drugs more recently described to cause erythema nodosum include gold and sulfonylureas.

•Enteropathies: Ulcerative colitis and Crohn disease may trigger erythema nodosum

•Hodgkin disease and lymphoma

•Sarcoidosis,Behcet’s syndrome,pregnancy,SLE

203. Which among the following is most frequently seen in anti phospholipid antibody syndrome?

A. Beta 2 microglobulin antibody

B. Anti nuclear antibody

C. Anti centromere antibody

D. Anti beta 2 glycoprotein antibody

Ans: D. Anti beta 2 glycoprotein antibody

Ref: Harrison

A large proportion of antiphospholipid antibodies are actually directed against complexes of phospholipid with protein, most notably beta-2-glycoprotein I. The ACA immunoassay detects both these clinically significant antibodies and antibodies directed purely against cardiolipin and not beta-2-glycoprotein I. The latter antibodies are transient, and are often associated with infection but not thrombosis. A positive anti-b2 GPI assay is evidence in favor of APS.

204. Ovoalbumin antigen was injected into a rabbit. What antibody will it produce initially?

A. IgG

B. IgM

C. IgE

D. IgD

Ans: B. IgM

Ref: Ananthanarayanan microbiology

“IgM is the main immunoglobulin produced early in the primary immune response. IgM is present on the surface of virtually all uncommitted B cells.”

Online support at www.pgpyrexia.co.cc

Free monthly AIIMS/ALL INDIA mock exams @ www.pgpyrexia.lefora.com.

Visit today!! Page 70

205. All are true about xanthogranulomatous inflammation except?

A. Presence of foamy macrophages

B. Presence of tuberculous infection

C. Multinucleated giant cell

D. Presence of yellow Nodules

Ans: B. Presence of tuberculous infection

Ref: Robbins

Xanthogranulomatous pyelonephritis is an unusual and relatively rare form of chronic pyelonephritis characterized by accumulation of foamy macrophages intermingled with plasma cells, lymphocytes, polymorphonuclear leukocytes, and occasional giant cells. Often associated with Proteus infections and obstruction, the lesions sometimes produce large, yellowish orange nodules that may be grossly confused with renal cell carcinoma.

206. Which among the following is the hallmark of acute inflammation?

A. Vasoconstriction

B. Stasis

C. Vasodilation and increase in permeability

D. Leukocyte margination

Ans: C. Vasodilation and increase in permeability

Ref: Robbins

A hallmark of acute inflammation is increased vascular permeability leading to the escape of a protein-rich exudate into the extravascular tissue, causing edema. Several mechanisms are responsible for the increased vascular permeability.

207. Which of the following helps in generating oxygen burst for killing bacteria within neurophils?

A. Superoxide dismutase

B. NADPH Oxidase

C. Peroxidase

D. Glutathione peroxidise

Ans: B. NADPH Oxidase [REPEAT]

208. Ophtalmoplegic migraine means?

A. Headache with reversible lose of ophthalmic nerve function

B. Recurrent transient 3rd nerve palsy after headache

C. Recurrent transient 3rd nerve palsy after headache always associated with central scotoma

D. Permanent 3rd nerve palsy following headache

Ans: B. Recurrent transient 3rd nerve palsy after headache [REPEAT]

Online support at www.pgpyrexia.co.cc

Free monthly AIIMS/ALL INDIA mock exams @ www.pgpyrexia.lefora.com.

Visit today!! Page 71

209. Which complement component is involved in both classical and alternate pathway?

A. C1

B. C2

C. C3

D. C4

Ans: C. C3 [Ref: Harrison]

210. HbH is seen in?

A. Deletion of 3 alpha gene

B. Deletion of all 4 alpha genes answer

C. Deletion of 3 beta genes

D. Deletion of all 4 beta genes

Ans: A. Deletion of 3 alpha genes

Ref: Harrison

Online support at www.pgpyrexia.co.cc

Free monthly AIIMS/ALL INDIA mock exams @ www.pgpyrexia.lefora.com.

Visit today!! Page 72

Alpha Thalassemia Syndromes:

The four classic alpha thalassemias, most common in Asians, are alpha-thalassemia-2 trait, in which one of the four alpha-globin loci is deleted; -thalassemia-1 trait, with two deleted loci; HbH disease, with three loci deleted; and hydrops fetalis with Hb Bart's, with all four loci deleted

211. True about prion ?

A. Catalyses folding of other proteins

B. Activates cytochromal enzymes

C. Activates apoptosis in neurons

D. Inhibits cytochrome p450 enzyme

Ans: A. Catalyses folding of other proteins

Ref: Jawetz microbiology

An abnormal isoform of this protein (PrPrsc) is the only known component of the prion and is associated with transmissibility. It has the same amino acid sequence as PrPc, but differs physically from the normal cellular isoform by its high -sheet content, its insolubility in detergents, its propensity to aggregate, and its partial resistance to proteolysis. It is believed that PrPres induces PrPc to fold or refold into the prion form.

212. Chimerism is associated with?

A. Monochorionic monoamniotic twins

B. Monochorionic dizygotic twins

C. Singleton pregnancy

D. Vanishing twin

Ans: B. Monochorionic dizygotic twins

“Journal Ref: Blood Chimerism in a Dizygotic Dichorionic Pregnancy”

“Blood chimerism in monochorionic twins conceived by induced ovulation: Case report”

Blood chimerism “is more common in monochorionic dizygotic twins and rare in dichorionic twins.Cases in which a monochorionic placenta occurs in a twin pregnancy,vascular anstomosis is well described and can lead to blood chimerism as well as twin twin transfusion syndrome.

213. Principle orgenelle involved in the execution of apoptosis is?

A. Nucleus

B. Lysosome

C. Mitochondria

D. Endoplasmic reticulum

Ans: C. Mitochondria

Ref: Robbins

Online support at www.pgpyrexia.co.cc

Free monthly AIIMS/ALL INDIA mock exams @ www.pgpyrexia.lefora.com.

Visit today!! Page 73

The mitochondrial pathway is the major mechanism of apoptosis in all mammalian cells, and its role in a variety of physiologic and pathologic processes is well established. This pathway of apoptosis is the result of increased mitochondrial permeability and release of pro-apoptotic molecules (death inducers) into the cytoplasm

214. Hybrids are obtained by using the following technique except-

A. Attaching inactive viral particle on cell membrane

B. Adding ethylene glycol

C. Applying a small electric current

D. Reducing the viscosity of the membrane

Ans: D. Reducing the viscosity of the membrane

Ref: Read below

• Ethylene glycol is one of nano-materials used to produce hybrids (confirmed) • "One configuration of a hybrid, called the parallel system, consists of a smallgasoline engine linked to a

compact direct-current electric motor " • " increase in the viscosity of the solution, which increases the liquid-side boundary layer mass transfer

resistance and partitioning of TCE(Trichloroethylene"-Membranes in clean technologies: Theory and practice: Volume 2 - Page 726 by Andrzej Kołtuniewicz, Enrico Drioli

215. Slide fixing in pathology done by-

A. Formaldehyde

B. Alcohol

C. Picric acid

D. Glutraldehyde

Ans: B. Alcohol

There are five major groups of fixatives, classified according to mechanism of action:

• Aldehydes

• Mercurials

• Alcohols

• Oxidizing agents

• Picrates

216. Which among the following is the most common tumour associated with neurofibromatosis in a child?

A. Juvenile myelomonocytic leukemia.

B. Acute lymphoblastic leukemia

C. Acute monocytic leukemia

D. Acute myeloid leukemia

Ans: A. Juvenile myelomonocytic leukemia.

Online support at www.pgpyrexia.co.cc

Free monthly AIIMS/ALL INDIA mock exams @ www.pgpyrexia.lefora.com.

Visit today!! Page 74

Ref: Nelson

Juvenile Chronic Myelogenous Leukemia

Patients with this disease do not have the Philadelphia chromosome that is characteristic of CML. Patients with JCML present with rashes, lymphadenopathy, and splenomegaly. JCML is rare, constituting <2% of all cases of childhood leukemia. Therapeutic reports are largely anecdotal. Patients with neurofibromatosis type 1 have a predilection for this type of leukemia. Stem cell transplantation offers the best opportunity for cure, but much less so than for classic CML.

217. Coarctation of aorta is most commonly associated with ?

A. Bicuspid aoric valve

B. PDA

C. VSD

D. ASD

Ans: A. Bicuspid aoric valve [REPEAT]

218. All are true regarding phagocytosis by protozoa except?

A. Amoeba and other unicellular organisms make their living out of it

B. Phagocytose particles of < 5 microns size

C. Phagocytose particles of > 5 microns size

D. Digestion occurs within phagolysosomes

Ans: B. Phagocytose particles of < 5 microns size

Ref:Jawetz microbiology

� Amebas feed by engulfing food particles with their pseudopodia. � Amastigotes reproduce in macrophage phagolysosomes, which normally have a pH of 4.5. � Size of many protozoans and their sexual & asexual forms is>5microns. � The promastigotes are phagocytosed by macrophages, and the acidity within the phagolysosome induces them

to transform into round amastigotes

219. All are true about Nesidioblastosis except ?

A. Hypoglycemic episodes are seen

B. Occurs in adults more than children

C. Histopathology shows hyperplasia of islet cells

D. Diazoxide is used for treatment

Ans: B. Occurs in adults more than children

Ref: Nelson

� Nesidioblastosis is a term for hyperinsulinemic hypoglycemia of infancy attributed to excessive function of pancreatic beta cells with an abnormal microscopic appearance

� The abnormal histologic aspects of the tissue included the presence of islet cell enlargement, islet cell dysplasia, beta cells budding from ductal epithelium, and islets in apposition to ducts

Online support at www.pgpyrexia.co.cc

Free monthly AIIMS/ALL INDIA mock exams @ www.pgpyrexia.lefora.com.

Visit today!! Page 75

� Diazoxide, which acts by opening K+ sensitive ATP channels ,inhibits pancreatic secretion of insulin, stimulates glucose release from the liver, and stimulates catecholamine release. (This effect is opposite that of the sulfonylurea drugs used in diabetes mellitus, which close the ATP channel.)

220. All of the following are neuronal tumours except?

A. Gangliocytoma

B. Ganglioglioma

C. Neuroblastoma

D. Ependymoma

Ans: D. Ependymoma

Ref: Robbins

Several types of CNS tumors contain mature appearing neurons(ganglion cell these might be complete population(GANGLIOCYTOMA) or sometimes it might be mixed with glial tissue ( GANGLIOGLIOMA).. others include cerebral neuroblastoma ,central neurocytoma and dysembryonic neuroepithelial tumor.

221. True about gastric carcinoma is?

A. Occult bleeding in stool is not seen

B. Associated with achlorhydria/hypochlorhydria.

C. Always squamous cell carcinoma

D. Radiosensitive

Ans: B. Associated with achlorhydria/hypochlorhydria.

Ref: Robbins:

Table 87-2 Nitrate-Converting Bacteria as a Factor in the Causation of Gastric Carcinomaa

Exogenous sources of nitrate-converting bacteria:

Bacterially contaminated food (common in lower socioeconomic classes, who have a higher incidence of the disease; diminished by improved food preservation and refrigeration)

? Helicobacter pylori infection

Endogenous factors favoring growth of nitrate-converting bacteria in the stomach:

Decreased gastric acidity

Prior gastric surgery (antrectomy) (15- to 20-year latency period)

Atrophic gastritis and/or pernicious anemia

? Prolonged exposure to histamine H2-receptor antagonists

222. Blood examination of a patient revealed low calcium, high phosphate and raised PTH. Which of the following investigations need not be done?

A. Urine microscopy

B. PTH reassessment

C. Vitamin D levels

Online support at www.pgpyrexia.co.cc

Free monthly AIIMS/ALL INDIA mock exams @ www.pgpyrexia.lefora.com.

Visit today!! Page 76

D. Serum alkaline phosphatase

Ans: A. Urine microscopy

Ref: Harrison

“Secondary hyperparathyroidism is caused by any condition that gives rise to chronic hypocalcemia, which in turn leads to compensatory overactivity of the parathyroid glands. Renal failure is by far the most common cause of secondary hyperparathyroidism, although several other diseases, including inadequate dietary intake of calcium, steatorrhea, and vitamin D deficiency, may also cause this disorder”

“Diagnosis is usually established by application of the PTH immunoassay, tests for vitamin D metabolites, and measurements of the urinary cyclic AMP response to exogenous PTH”

OBSTRETICS AND GYNAECOLOGY:

223. Clue Cells are seen in :

A. Bacterial vaginosis

B. Vaginal candidiasis

C. Chlamydial vaginosis

D. Trichomoniasis

Ans: A. Bacterial vaginosis [REPEAT]

224. Fallopian tube immotility is seen in?

A. Churg strauss syndrome

B. Kartagener's syndrome

C. Noonan syndrome

D. Turner syndrome

Ans: B. Kartagener's syndrome [REPEAT]

225. Best treatment option for genuine stress incontinence?

A. Burch colposuspension

B. Kelly’s procedure

C. Sling operation

D. Tension free vaginal taping

Ans: A. Burch colposuspension

Ref: Novak’s Gynaecology

Burch Procedure:

Two pairs of large-caliber delayed-absorbable suture are placed through the periurethral vaginal wall, one pair at the midurethra and one at the urethrovesical junction. Each stitch is then anchored to the ipsilateral Cooper's (iliopectineal) ligament. The sutures are tied with the operator's nondominant hand placed vaginally to give preferential support to the

Online support at www.pgpyrexia.co.cc

Free monthly AIIMS/ALL INDIA mock exams @ www.pgpyrexia.lefora.com.

Visit today!! Page 77

urethrovesical junction relative to the anterior vaginal wall without overcorrection. Long-term outcome studies up to 10 years have shown the Burch procedure yields cure rates of 80 to 85%. [Ref: Schwartz]

In 1997, the American Urological Association convened a clinical guidelines panel to analyze published outcomes data on surgical procedures to treat female stress urinary incontinence and to produce practice recommendations to guide surgical decision–making . The panel concluded that colposuspensions (e.g., Burch, Marshall–Marchetti–Krantz [MMK]) and slings were more effective than transvaginal needle suspensions or anterior repairs for long–term success (48 month cure/dry rates). The median probability estimates for cure/dry rates at 48 months and longer were 84% (95% CI, 79%–88%) for colposuspensions and 83% (95% CI, 75%–88%) for sling procedures, compared with 67% (95% CI, 53%–79%) for transvaginal needle suspensions and 61% (95% CI, 47%–72%) for anterior repairs.

226. All of the following are done in management of shoulder dystocia except?

A. Fundal pressure

B. Suprapubic pressure

C. McRoberts maneuver

D. Woods maneuver

Ans: A. Fundal pressure [REPEAT]

227. Which of the following is not a contraindication for pregnancy?

A. WPW syndrome

B. Pulmonary hypertension

C. Eisenmenger syndrome

D. Marfan syndrome with aortic root dilatation

Ans: A. WPW syndrome [REPEAT]

228. Weight gain in pregnancy is related to all except?

A. Ethnicity

B. Smoking

C. Socioeconomic status

D. Pre conceptional weight

Ans: B. Smoking [REPEAT]

229. Best marker for intrahepatic cholestasis of pregnancy is?

A. Bile salts

B. Bile acid

C. Bilirubin

D. Alkaline phosphatase

Ans: B. Bile acid [REPEAT]

Online support at www.pgpyrexia.co.cc

Free monthly AIIMS/ALL INDIA mock exams @ www.pgpyrexia.lefora.com.

Visit today!! Page 78

230. Mifepristone is used in?

A. Molar pregnancy

B. Threatened abortion

C. Fibroid

D. Ectopic pregnancy

Ans: C. Fibroid >> other options

Ref: Goodman & Gillman

Therapeutic Uses and Prospects.

Mifepristone (MIFEPREX), in combination with misoprostol or other prostaglandins (see below), is available for the termination of early pregnancy. When mifepristone is used to produce a medical abortion, a prostaglandin is given 48 hours after the anti-progestin to further increase myometrial contractions and ensure expulsion of the detached blastocyst.

Other investigational or potential uses for mifepristone that are under development include the induction of labor after fetal death; the induction of labor at the end of the third trimester; treatment of endometriosis, leiomyomas, breast cancer, and meningiomas; and as a postcoital or luteal-phase contraceptive (Spitz and Chwalisz, 2000).

231. Causes of primary amenorrhoea are all except?

A. Rokintasky syndrome

B. Kallaman syndrome

C. Sheehan syndrome

D. Turner syndrome

Ans: C. Sheehan syndrome [REPEAT]

232. Which of the following are not associated with menstrual cycle in humans?

A. Hormonal changes

B. Vaginal cytology changes

C. Estrus profile

D. Endometrial changes

Ans: C. Estrus profile [REPEAT]

233. A 45 year old lady presented with DUB & USG finding of 8mm thick endometrium. What is the next step?

A. Endometrial histopathology

B. Hysterectomy

C. OCP

D. Follow up

Ans: A. Endometrial histopathology [REPEAT]

Online support at www.pgpyrexia.co.cc

Free monthly AIIMS/ALL INDIA mock exams @ www.pgpyrexia.lefora.com.

Visit today!! Page 79

234. XO genotype commonly associated with?

A. Gonadal dysgenesis

B. Gonadal agenesis

C. Female pseudohermaphroditism

D. Male peudohermaphroditism

Ans: A. Gonadal dysgenesis

Ref: Robbins

Half the patients with Turner syndrome have a 45,X chromosomal complement. About 15% of patients are mosaics for 45,X and a normal cell line (45,X/46,XX). Other mosaics with isochromosomes, 45,X/46,X,i(Xq); with rings, 45,X/46,X,r(X); or with fragments, 45,X/46fra, occur less often. Mosaicism is detected most commonly when more than one tissue is examined. The single X is of maternal origin in nearly 80% of 45,X patients.

235. Open NTD are best diagnosed by raised?

A. Acetylcholinesterase

B. Pseudocholinesterase

C. Amniotic fluid

D. Nuchal fold thickness

Ans: A. Acetylcholinesterase [REPEAT]

236. Earliest to be diagnosed by USG is?

A. Anencephaly

B. Prosencephaly

C. Meningocele

D. Spina bifida

Ans: A. Anencephaly (9-11 weeks) [REPEAT]

237. Regarding PCOD, all are true except?

A. High LH/FSH

B. High DHEAS

C. Very high prolactin

D. Raised LH

Ans: C. Very high prolactin [REPEAT]

Online support at www.pgpyrexia.co.cc

Free monthly AIIMS/ALL INDIA mock exams @ www.pgpyrexia.lefora.com.

Visit today!! Page 80

238. Which drug is not in the conduct of labour in a woman with rheumatic heart disease?

A. Methylergometrine

B. Carboprost

C. Syntocin

D. Misoprostol

Ans: A. Methylergometrine [REPEAT]

239. Least common cause of ambiguous genitalia in a female child?

A. Placental steroid sulfatase deficiency

B. Fetal aromatase deficiency

C. WT-1 mutation

D. CAH

Ans: A. Placental steroid sulfatase deficiency

MULTIPLE SULFATASE DEFICIENCY:

This is an autosomal recessive disorder that results from the deficiency of three enzymatic activities: arylsulfatases A, B, and C. Sulfatides, mucopolysaccharides, steroid sulfates, and gangliosides accumulate in the cerebral cortex and visceral tissues, resulting in a clinical phenotype with features of leukodystrophy as well as those of the mucopolysaccharidoses. Severe ichthyosis may also occur. Carrier testing and prenatal diagnosis by measurement of the enzymatic activities can be performed. There is no specific treatment for multiple sulfatase deficiency other than supportive care.

Online support at www.pgpyrexia.co.cc

Free monthly AIIMS/ALL INDIA mock exams @ www.pgpyrexia.lefora.com.

Visit today!! Page 81

240. Which of the following is not an evidence based treatment for menorrhagia?

A. Ethamsylate

B.OCP

C.Tamoxifene

D.Tranaxemic acid

Ans: A. Ethamsylate [REPEAT]

DERMATOLOGY:

241. Cicatrising alopecia with perifollicular greying is most commonly associated with?

A. Nail dystrophy

B. Whitish lesion in the buccal mucosa

C. Arthritis

D. Discoid Plaques in the face

Ans: B. Whitish lesion in the buccal mucosa

Ref: Rook’s Dermatology

Lichen planus occurs at any age, but in over 80% of cases the onset is between 30 and 70 years. Significant involvement of the scalp is relatively infrequent—only 10 of 807 patients in one series —but the incidence is probably rather higher than such figures suggest, because they tend to exclude those patients in whom alopecia, classified as pseudopelade, was the only manifestation of the disease.

Follicular lesions occurring in the scalp are accompanied by some scaling and are likely to lead to a scarring alopecia. Very rarely, the scalp alone is involved. The so-called

Graham Little–Piccardi–Lassueur syndrome comprises the triad of multifocal scalp cicatricial alopecia, non-scarring alopecia of the axillae and/or groin, and keratotic lichenoid follicular papules.

Mucous membrane lesions are very common, occurring in 30–70% of cases, and may be present without evidence of skin lesions. The buccal mucosa and tongue are most often involved, but lesions may be found around the anus, on the genitalia, in the larynx and, very rarely, on the tympanic membrane of the ears or even in the oesophagus. White streaks, often forming a lacework, on the buccal mucosa are highly characteristic. They may be seen on the inner surface of the cheeks, on the gum margins or on the lips.

242. A man presents with a maculopapular rash. He gives a history of previous painless rash. Infection is due to?

A. Treponema pallidum

B. Chlamydia

C. Calymmatobacterium granulomatis

D. Haemophilus ducreyi

Ans: A. Treponema pallidum

Ref: Robbins

Online support at www.pgpyrexia.co.cc

Free monthly AIIMS/ALL INDIA mock exams @ www.pgpyrexia.lefora.com.

Visit today!! Page 82

Primary Syphilis.

This stage, occurring approximately 3 weeks after contact with an infected individual, features a single firm, nontender, raised, red lesion (chancre) located at the site of treponemal invasion on the penis, cervix, vaginal wall, or anus. The chancre heals in 3 to 6 weeks with or without therapy.

Secondary Syphilis.

This stage usually occurs 2 to 10 weeks after the primary chancre and is due to spread and proliferation of the spirochetes within the skin and mucocutaneous tissues. Secondary syphilis occurs in approximately 75% of untreated people. The skin lesions, which frequently occur on the palms or soles of the feet, may be maculopapular, scaly, or pustular. All these painless superficial lesions contain spirochetes and so are infectious.

243. Pseudoisomorphic phenomenon seen in

A. Psoriasis

B. Lichen planus

C. Vitiligo

D. Plane warts

Ans: D. Plane warts

Ref: Rook’s Dermatology

Koebner response is best not used when a dermatosis occurs resulting from the spread of an infective agent (e.g. molluscum contagiosum or warts); for this phenomenon, the term pseudo-Koebner could be used.

ANAESTHESIA:

244. Anesthetic agent with vasoconstrictor is contraindicated in?

A. Finger block

B. Spinal block

C. Epidural block

D. Regional anaesthesia

Ans: A. Finger block

Ref: Morgan’s Anasthesia

Digital Nerves: These nerve blocks are used for minor operations on the fingers and to supplement brachial plexus blocks.

TECHNIQUE: A 23-to25-gauge needle is inserted at the medial and lateral aspects of the base of the selected digit. A total of 2–3 mL of local anesthetic without epinephrine is injected on each side near the periosteum. Addition of a vasoconstrictor (epinephrine) can seriously compromise blood flow to the digit.

245. Which anesthetic modality is to be avoided in sickle cell disease?

A. General anesthesia

B. Brachial plexus block

C. IV Regional Anesthesia

Online support at www.pgpyrexia.co.cc

Free monthly AIIMS/ALL INDIA mock exams @ www.pgpyrexia.lefora.com.

Visit today!! Page 83

D. Spinal

Ans: C. IV Regional Anesthesia

Ref: Morgan’s Anasthesia

Intravenous Regional Sympathetic Blockade

A Bier block utilizing guanethidine (20–40 mg) can selectively interrupt sympathetic innervation to an extremity. Ten milliliters of lidocaine 0.5% can also be added to prevent burning. A tourniquet is placed proximally on the extremity and usually left inflated for at least 20 min. Guanethidine causes depletion of norepinephrine and inhibits its reuptake at the terminals of postganglionic neurons.

Conditions that might promote hemoglobin desaturation or low-flow states should be avoided. Every effort must be made to avoid hypothermia and hyperthermia, acidosis, and even mild degrees of hypoxemia, hypotension, or hypovolemia. Generous hydration and a relatively high (> 50%) inspired oxygen tension are desirable. The major compensatory mechanism in these patients is an increased cardiac output, which should be maintained intraoperatively. Monitoring central venous pressure or pulmonary artery pressure with mixed venous oxygen saturation may be useful in some patients. Mild alkalosis may help avoid sickling, but even moderate degrees of respiratory alkalosis may have an adverse effect on cerebral blood flow. Many clinicians will also avoid the use of tourniquets

246. Definitive airway is all except?

A. Nasotracheal tube

B. Orotracheal tube

C. LMA

D. Cricothyroidotomy

Ans: C. LMA

Ref: MILLER’s Anesthesia

THE DEFINITIVE AIRWAY: A definitive airway can be: an endotracheal tube, an nasotracheal tube, or a surgical airway (cricothroidotomy).

Elective use of an LMA in a patient with a known or anticipated difficult airway has serious disadvantages. The difficult airway remains and the development of airway obstruction could produce a critical situation that requires immediate percutaneous airway rescue. Asai states, “It is inadvisable to rely on the LMA when tracheal intubation is predicted to be difficult.”

247. Epileptic potential is present in

A. Desflurane

B. Halothane

C. Sevoflurane.

D. Ether

Ans: C. Sevoflurane.

Ref:MILLER’s Anesthesia

Epileptiform activity has been induced by administration of sevoflurane in patients without epilepsy, and seizure activity on EEG, but not clinical seizure activity, has been reported in pediatric patients with a history of epilepsy during induction of anesthesia with sevoflurane.

Online support at www.pgpyrexia.co.cc

Free monthly AIIMS/ALL INDIA mock exams @ www.pgpyrexia.lefora.com.

Visit today!! Page 84

248. True about epidural opioids are all except?

A. Acts on dorsal horn substantia gelatinosa

B. Can cause Itching

C. Function of the intestines are not affected

D. Can cause respiratory depression

Ans: C. Function of the intestines are not affected

Ref: Miller’s Anesthesia

When compared with systemic opioids, perioperative epidural analgesia may confer several advantages, including a facilitated return of gastrointestinal function and decrease in the incidence of pulmonary complications, coagulation-related adverse events, and possibly cardiovascular events, especially in higher-risk patients or procedures

249. Spinal anaesthesia is given at which level?

A. L1-2

B. L2-4

C. S1

D. Thoracic segment

Ans: B. L2-4

Ref: Miller’s Anesthesia

After the equipment, local anesthetics and additives, and the patient have been properly prepared, the midline or paramedian spinal puncture can be performed. The midline approach relies on the ability of patients and assistants to minimize lumbar lordosis and allow access to the subarachnoid space between adjacent spinous processes, usually at the L2-3, L3-4, or sometimes the L4-5 space.

250. An anesthesia resident was giving spinal anaesthesia when the patient had sudden aphonia and loss of consciousness. What could have happened?

A. Total spinal

B. Partial spinal

C. Vaso vagal attack

D. Intra vessel injection

Ans: A. Total spinal

Ref: Morgan’s Anesthesia

Spinal anesthesia ascending into the cervical levels causes severe hypotension, bradycardia, and respiratory insufficiency. Unconsciousness, apnea, and hypotension resulting from high levels of spinal anesthesia are referred to as a "high spinal" or "total spinal." It can also occur following attempted epidural/caudal anesthesia if there is inadvertent intrathecal injection (see below). Severe sustained hypotension with lower sensory blocks can also lead to apnea through medullary hypoperfusion.

251. Which of the following anesthetic drugs is contraindicated in a patient with hypertension?

A. Ketamine

B. Propofol

C. Etomidate

Online support at www.pgpyrexia.co.cc

Free monthly AIIMS/ALL INDIA mock exams @ www.pgpyrexia.lefora.com.

Visit today!! Page 85

D. Diazepam

Ans: A. Ketamine

Ref: Miller’s Anesthesia

Ketamine also has unique cardiovascular effects; it stimulates the cardiovascular system and is usually associated with increases in blood pressure, heart rate, and cardiac output.

252. A 40 year old female underwent surgery. Post operatively she told the anaesthetist that she was aware of per-operative events. Individual intraoperative awareness is evaluated by (to prevent such instances from occurring)?

A. Pulse oximetry

B. Colour doppler

C. Bispectral imaging

D. End tidal CO2

Ans: C. Bispectral imaging

Ref: Morgan’s Anesthesia

New two-channeled processed EEG devices pass the EEG signal through a fast Fourier transform (bispectral analysis) leading to a traditional power spectrum. The Bispectral Index (BIS) represents a numerical value that has been correlated with the patient's current hypnotic state. It is currently popular among some to rely upon the index derived from bispectral analysis of the electroencephalogram to assess depth of anesthesia and to prevent awareness.

PSYCHIATRY:

253. A schizophrenic patient started on haloperidol 2 days back, comes with complaints of torticollis and orofaciolingual movements. What is the diagnosis?

A. Acute dystonia

B. Tardive dyskinesia

C. Parkinsonism

D. Akathisia

Ans: A. Acute dystonia

Antipsychotic medications can cause a broad range of side effects, including lethargy, weight gain, postural hypotension, constipation, and dry mouth. Extrapyramidal symptoms such as dystonia, akathisia, and akinesia are also frequent with first-generation agents and may contribute to poor adherence if not specifically addressed.

Neuroleptic agents also exert characteristic neurological effects including bradykinesia, mild rigidity, tremor, and subjective restlessness (akathisia) that resemble the signs of Parkinson's disease.

254. All are true about delirium tremens except?

A. Visual hallucinations

B. Coarse tremors

C. Ocular palsy

D. Unconsciousness

Online support at www.pgpyrexia.co.cc

Free monthly AIIMS/ALL INDIA mock exams @ www.pgpyrexia.lefora.com.

Visit today!! Page 86

Ans: C. Ocular palsy

Ref: Kaplan and Sadock Synopsis of Psychiatry

The essential feature of the syndrome is delirium occurring within 1 week after a person stops drinking or reduces the intake of alcohol. In addition to the symptoms of delirium, the features of alcohol intoxication delirium include autonomic hyperactivity such as tachycardia, diaphoresis, fever, anxiety, insomnia, and hypertension; perceptual distortions, most frequently visual or tactile hallucinations; and fluctuating levels of psychomotor activity, ranging from hyperexcitability to lethargy.

255. A female patient presented with depressed mood, loss of appetite and no interest in surroundings. There is associated insomnia. The onset of depression was preceeded by a history of business loss. What is the line of management?

A. No treatment is necessary as it is due to business loss

B. SSRI is the best choice

C. Start SSRI treatment based on side effect profile

D. Combination therapy of 2 anti depressant drugs

Ans: C. Start SSRI treatment based on side effect profile

Ref: Kaplan and Sadock Synopsis of Psychiatry

Initial Medication Selection

The available antidepressants do not differ in overall efficacy, speed of response, or long-term effectiveness. Antidepressants, however, do differ in their pharmacology, drug-drug interactions, short- and long-term side effects, likelihood of discontinuation symptoms, and ease of dose adjustment. Failure to tolerate or to respond to one medication does not imply that other medications will also fail. Selection of the initial treatment depends on the chronicity of the condition, course of illness (a recurrent or chronic course is associated with increased likelihood of subsequent depressive symptoms without treatment), family history of illness and treatment response, symptom severity, concurrent general medical or other psychiatric conditions, prior treatment responses to other acute phase treatments, potential drug–drug interactions, and patient preference.

In general, approximately 45 to 60 percent of all outpatients with uncomplicated (i.e., minimal psychiatric and general medical comorbidity), nonchronic, nonpsychotic major depressive disorder who begin treatment with medication respond (i.e., achieve at least a 50 percent reduction in baseline symptoms); however, only 35 to 50 percent achieve remission (i.e., the virtual absence of depressive symptoms).

256. Regarding an imbecile, all are true except?

A. IQ is 50-60

B. Intellectual capacity equivalent to a child of 3-7 years of age

C. Not able to take care of themselves

D. Condition is congenital or acquired at an early age.

Ans: A. IQ is 50-60

IQ Range Classification

70-80 Borderline deficiency

50-69 Moron

20-49 Imbecile

below 20 Idiot

Online support at www.pgpyrexia.co.cc

Free monthly AIIMS/ALL INDIA mock exams @ www.pgpyrexia.lefora.com.

Visit today!! Page 87

RADIOLOGY

257. How to differentiate ASD from VSD in X-ray?

A. Enlarged Left atrium

B. Normal left atrium.

C. Pulmonary congestion

D. Aortic shadow

Ans: B. Normal left atrium.

Ref: Essentials of Radiology by Fred A. Mettler

Acyanotic congenital heart disease similarly should initially be evaluated by determination of pulmonary vascularity. In those with normal vascularity, aortic stenosis, pulmonic stenosis, coarctation, and interruption of the aortic arch should be considered. Acyanotic heart disease with increased pulmonary vascularity should next be investigated by looking for left atrial enlargement. This is not present in ASD, and an endocardial cushion defect may be considered.

An ASD is the most common congenital cardiac anomaly in adults and rarely is symptomatic in infancy or childhood. The common radiologic findings in an ASD, in addition to enlargement of the pulmonary artery, are an increase in the size of the right atrium and right ventricle. This is often best seen as filling in of the retrosternal clear space on the lateral view

258. Aortic knuckle shadow on chest X ray, PA view is obliterated by consolidation of which portion of left lung?

A. Upper lingula

B. Lower lingula

C. Apex of lower lobe

D. Posterior part of upper lobe

Ans: D. Posterior part of upper lobe

Ref: Sutton’s Radiology

Silhouette sign:

Definition: Loss of density contrast borders due to increased density of another tissue. If an infiltrate is large enough to occupy a significant part of a lobe it may result in the disappearance of the silhouette of an adjacent structure. When lung tissue is replaced by blood, pus, or tumor, the radiographic outlines of the mediastinal or diaphragmatic organs superimposed on these infiltrated areas will disappear.

Remember this,

The right middle lobe and lingula lie adjacent to the right and left cardiac borders, the apicoposterior segment of the left upper lobe lies adjacent to the aortic knuckle

So a consolidation of Posterior part of upper lobe of left lung will obliterate Aortic knuckle.

259. CT scan is least accurate for diagnosis of:

A. 1 cm size aneurysm in an artery

B. 1 cm size lymph node in thyroid carcinoma

Online support at www.pgpyrexia.co.cc

Free monthly AIIMS/ALL INDIA mock exams @ www.pgpyrexia.lefora.com.

Visit today!! Page 88

C. 1 cm size mass in tail of pancreas

D. 1 cm size gall stone

Ans: D. 1 cm size gall stone

Ref: Review of Radiology by Sumer Sethi

<10% of gallstones and >90% of renal stones are radioopaque.

Best IOC for Cholelithiasis is Tc99-HIDA Scan.

260. Dose of radiation required for development of haematological syndrome is?

A. 2.5-5 Gy

B. 10 Gy

C. 100 cGy

D. 200 cGy

Ans: C. 100 cGy

Threshold dosage for Acute Radiation syndrome of haematological origin following a whole body irradiation – 1 Gy [=100cGy]

[Ref: Review of Radiology by Sumer Sethi]

Online support at www.pgpyrexia.co.cc

Free monthly AIIMS/ALL INDIA mock exams @ www.pgpyrexia.lefora.com.

Visit today!! Page 89

261. About contrast radiography true is (HIGHLY UNCLEAR ON OPTIONS)

A. 3 parts of iodine with 2 parts of solvent

B. Injection into artery is associated with 1/3 times more complication than injection into a vein

C. Test dose should be performed if a contrast reaction is suspected

D. Not recalled

Ans: C.Test dose should be performed if a contrast reaction is suspected

MEDICINE AND MISCALLENOUS:

262. ECG is poor at detecting ischaemia in areas supplied by?

A. Left anterior descending

B. Left circumflex

C. Left coronary artery

D. Right coronary artery

Ans: B. Left circumflex

Direct line pick from: Dynamic electrocardiography By Marek Malik, A. John Camm

“The standard 12-lead ECG is less sensitive in detecting occlusion of the LCx coronary artery than it is in detecting one in the other arteries”

Reason:

The left circumflex coronary artery subtends the lateral and posterior walls of the left ventricle.While the lateral wall is represented in leads V6 and aVL,the posterior wall is poorly represented by the standard 12-lead ECG

Also note:

“The ECG is most sensitive to detect acute occlusion of the LAD and is least sensitive for involvement of the circumflex artery.”

263. A girl presented with occipital headache associated with ataxia and vertigo. Mother also has similar complaints. Most probable diagnosis is?

A. Cervical vertigo

B. Multiple sclerosis

C. Basilar migraine

D. Vestibular neuronitis

Ans: C. Basilar Migraine

Ref: CMDT 2010 Pg.190, Pl Dhingra 3rd ed, Pg.59

Rule out the options:

A) Cervical vertigo-- triggered by the neck movements due to cervical proprioceptive dysfunction.It is due to Neck injury particularly Hyper-extension. and degenerative cervical spine disease.Vertigo triggered by particular head position.

No mention about the ataxia and familial predisposition has been made.(CMDT 2010 Pg.190)

Online support at www.pgpyrexia.co.cc

Free monthly AIIMS/ALL INDIA mock exams @ www.pgpyrexia.lefora.com.

Visit today!! Page 90

B) Multiple sclerosis: All the features excluding headache are mentioned even the familial inheritancy. Trigeminal neuralgia, Glossopharyngeal neuralgia Hemifacial spasm occurs in MS. Harrison 17th ed

C)Basilar migraine also known as Bickerstaff syndrome.Most frequent in adeloscent girls and young women.Features are Headache, accompanied by dizziness, vertigo, decreased hearing ,nausea, vomiting, syncope etc.It is due to the autosomal dominant mutation of ATP1A2 gene. (e-medicine: Diff types of migrain)

D)Vestibular neuronitis: Paroxysmal vertigo episode with out cochlear symptoms.It occurs due to viral infection of vestibular ganglion.(Pl Dhingra 3rd ed, Pg.59)

264. All are false with respect to PSUDOHYPOPARATHYROIDISM except?

A. Decreased cAMP response

B. Decreased IP3 response

C. Increased GTPase activity

D. Gain of function mutation of Gs alpha subunit

Ans: A. Decreased cAMP

Reference: Harrison Principles Of Internal Medicine17th ed

Individuals with Pseudohypoparathyroidism 1, the most common of the disorders, show a deficient urinary cyclic AMP response to administration of exogenous PTH.

Patients with PHP-I are divided into type a, with Albright’s Hereditary Osteodystrophy(AHO) and reduced amounts of Gsalpha in vitro assays witherythrocytes, and type b, lacking AHO and with normal amounts of Gsalpha in erythrocytes. There is a third type (PHP-Ic, reported in a few patients) that differs from PHP-Ia only in having normal erythrocyte levels of Gsalpha despite having AHO, hypocalcemia, and decreased urinary cyclic AMP responses to PTH (presumably with a post-Gsalpha defect in adenyl cyclase stimulation).

265. A 1yr old child present with growth failure,dry skin and palpable thyroid,with low thyroid harmones and a high TSH, what is cause?

A. Dysharmonogenesis

B. Dysgenesis

C. Central Hypothyroidism

D. TSH Receptor mutation

Ans: B.Dysgenesis [REPEAT]

266. About diabetes insipidus all true except

A.Water deprivation test is diagnostic.

B. Before doing test first correct hypoaldosteronism.

C. Hypothyroidism not affect the diagnostic test.

D. Pre test serum osmolarity >288, before giving vasopressin.

Ans: C. Hypothyroidism not affect the diagnostic test.

This is an answer by exclusion

Online support at www.pgpyrexia.co.cc

Free monthly AIIMS/ALL INDIA mock exams @ www.pgpyrexia.lefora.com.

Visit today!! Page 91

Opt A. The water deprivation test is used to differentiate between CDI and NDI in a patient likely to have one or the other.so,it is very clear that it is a diagnostic test.

Opt B.This is also true since in ypoaldosteronism there is extravascular sodium loss which depletes ECF volume which in turn leads to increaded osmolarity which can give false values in water depletion test

Opt.D.Harrison states “if the 24-h urine osmolarity is <300 mosmol/L, the patient has a water diuresis and should be evaluated further to determine which type of DI is present”. so this option is also true

267. A 35 year old female has proximal weakness of muscles, ptosis and easy fatiguability. The best test to diagnose her condition is:

A. Muscle biopsy

B. CPK

C. Edrophonium test

D. EMG

Ans: C. Edrophonium test [REPEAT]

268. The acid base status of a patient is as follows: pH - 7.45, pCO2 - 30 mm of Hg, pO2 - 105 mm of Hg. Patient has partially compensated?

A. Metabolic acidosis

B. Metabolic alkalosis

C. Respiratory acidosis

D. Respiratory alkalosis

Ans: D. Respiratory alkalosis

*** pH indicates alkalosis.if Pco2 is less than normal it indicates respiratory cause and thus the answer.

269. In uncompentated Diabetic ketoacidosis not true is?

A. pco2 levels are normal or increased

B. Increased levels of lactate

C. Decreased HCO3

D. Not recalled

Ans: C. Decreased HCO3

Table from Harrison (very important)

Online support at www.pgpyrexia.co.cc

Free monthly AIIMS/ALL INDIA mock exams @ www.pgpyrexia.lefora.com.

Visit today!! Page 92

270. Which among the following not used in diagnosis of insulinoma?

A. Fasting glucose test

B. Xylulose test

C. C peptide levels

D. Insulin / glucose ratio

Ans: B. Xylulose test

(It is a test for malabsorption)

271. Intraoperative myocardial infarction is best diagnosed by:

A. ECG

B. Invasive arterial pressure

C. Central venous pressure

D. Trans-esophageal echo

Ans: D. Trans-esophageal echo [REPEAT]

“The improved quality of the acoustic image enabled anesthesiologists and surgeons to use TEE intraoperatively to diagnose myocardial ischemia, confirm the adequacy of valve reconstruction and other surgical repairs, determine the cause of hemodynamic disorders and other intraoperative complications, and provide diagnostic information that could not be obtained preoperatively.”

Online support at www.pgpyrexia.co.cc

Free monthly AIIMS/ALL INDIA mock exams @ www.pgpyrexia.lefora.com.

Visit today!! Page 93

272. All are true about pheochromocytoma except?

A. 90% are malignant

B. 95% occur in the abdomen

C. They secrete catecholamines

D. They arise from sympathetic ganglions

Ans: A. 90% are malignant

Ref: Harrison

The "rule of ten" for pheochromocytomas states that about

• 10% are bilateral, • 10% are extraadrenal, • 10% in children • 10% familial • 10% are malignant.

However, these percentages are higher in the inherited syndromes.

273. Which of the following is not included in parenteral nutrition?

A. Fat

B. Carbohydrate

C. Fibre

D. Micronutrients

Ans: C. Fibre

TPN requires water (30 to 40 mL/kg/day), energy (30 to 60 kcal/kg/day, depending on energy expenditure), amino acids (1 to 2.0 g/kg/day, depending on the degree of catabolism), essential fatty acids, vitamins, and minerals.

275. A young lady presents with fever, dysuria and pain abdomen. Uncomplicated acute cystitis was diagnosed. Which among the following is not true?

A. Nitrite dipstick test positive

B. E.coli count was < 10^3

C. 1 pus cell per 7 fields

D. 1 bacillus per field

Ans: C. 1 pus cell per 7 fields

Ref: Manual of Nephrology: Diagnosis and Therapy By Robert W. Schrier

Let’s come to the other by discussing options one by one:

Opt.A: In today's office practice, the dipstick test for nitrite is used as a surrogate marker for bacteriuria

Opt.B: The diagnosis of UTI was based on a quantitative urine culture yielding greater than 100,000 colony-forming units (105 CFU) per milliliter of urine, which was termed "significant bacteriuria." This value was chosen because of its high specificity for the diagnosis of true infection, even in asymptomatic persons. However, several studies have established that 30% or more of symptomatic women have CFU counts below this level (low-coliform-count infections). They have

Online support at www.pgpyrexia.co.cc

Free monthly AIIMS/ALL INDIA mock exams @ www.pgpyrexia.lefora.com.

Visit today!! Page 94

also shown that a bacterial count of 100 CFU/ml of urine has a high positive predictive value for cystitis in symptomatic women .

(So option B is also true)

Opt.C: In a patient with Cystitis with symptoms,it is expected that atleast 1 pus cell be present in every oil immersion field.

276. Which of the following is not an adverse effect of thalidomide?

A. Diarrhoea

B. Teratogenicity

C. DVT

D. Peripheral neuropathy

Ans: A. Diarrhea

Reference: Goodman and Gilman Pharmacology

The most common adverse effects reported in cancer patients are sedation and constipation, while the most serious one is treatment-emergent peripheral sensory neuropathy, which occurs in 10% to 30% of patients with MM or other malignancies in a dose- and time-dependent manner. Thalidomide-related neuropathy is an asymmetric, painful, peripheral paresthesia with sensory loss, commonly presenting with numbness of toes and feet, muscle cramps, weakness, signs of pyramidal tract involvement, and carpal tunnel syndrome

277. Cause of premature death in schizophrenia?

A. Homicide

B. Suicide

C. Toxicity of antipsychotic drug

D. Hospital acquired infection

Ans: B. Suicide

Ref: Kaplan and Saddock

Suicide is the single leading cause of premature death among people with schizophrenia. Suicide attempts are made by 20 to 50 percent of the patients, with long-term rates of suicide estimated to be 10 to 13 percent. These numbers reflect an approximately 20-fold increase over the suicide rate in the general population. Often, suicide in schizophrenia seems to occur out of the blue without prior warnings or expressions of verbal intent. The most important factor is the presence of a major depressive episode.

278. Most important and potential agent that can be used in bioterrorism:

A. Plague

B. Small pox

C. TB

D. Clostridium botulinum

Ans: D. Clostridium botulinum

According to Harrison,other than TB all other optons are mentioned in HIGH-PRIORITY CLASS A agents

Online support at www.pgpyrexia.co.cc

Free monthly AIIMS/ALL INDIA mock exams @ www.pgpyrexia.lefora.com.

Visit today!! Page 95

We choose Cl.botulinum as Ans in view of folloing lines from Harrison:

“Unique among the category A agents for not being a live microorganism, botulinum toxin is one of the most potent toxins ever described and is thought by some to be the most poisonous substance in existence”

279. Rise in end tidal CO2 during thyroid surgery can be due to all except:

A. Anaphylaxis

B. Malignant hyperthermia

C. Thyroid storm

D. Neuroleptic malignant syndrome

Ans: A. Anaphylaxis

280. A 15 day old baby came with history of seizures. Blood tests revealed Ca 5mg/dl, PO4 9mg/dl, PTH 30pg/ml (n=10-60). What is the most probable diagnosis?

A. Pseudohypoparathyroidism

B. Vitamin D deficiency

C. Hyperparathyroidism

D. HIE

Ans: A. Pseudohypoparathyroidism

Neonates with pseudohypoparathyroidism produce right amoun of PTH, but body is resistant to its effect. This causes low calcium level and high phosphate level.

This a case of a late onset seizures (>1 week) and hypocalcemia is well known to cause seizures at this time

Please refer Q no.143 for more details

Online support at www.pgpyrexia.co.cc

Free monthly AIIMS/ALL INDIA mock exams @ www.pgpyrexia.lefora.com.

Visit today!! Page 96

281. In expectant management of placenta praevia, all are done except?

A. Cervical encirclage

B. Anti D

C. Corticosteroids

D. Blood transfusion

Ans: A Cervical encirclage

Reference: Williams Obstetrics

The treatment of classical cervical incompetence is cerclage. The operation is performed to surgically reinforce the weak cervix by some type of purse-string suturing. Bleeding, uterine contractions, or ruptured membranes are usually contraindications for cerclage.

282. Most common nerve injured in supracondylar fracture humerus?

A. Median

B. Radial

C. Ulnar

D. Anterior interosseus nerve

Ans: D. Anterior interosseus nerve

Ref: The elbow and its disorders By Bernard F. Morrey, Joaquin Sanchez-Sotelo, Page 226,Skeletal trauma in children, Volume 3 By Neil E. Green, Marc F. Swiontkowski, Page 212

� Nerve injuries occur in about 40% of type III (Gartland’s classification) supracondylar fractures � Earlier literature stated that radial nerve was the most commonly injured nerve in supracondylar fractures � But recent studies indicate that the anterior interosseous branch of median nerve is mostly affected � Nerve involvement differ with the type of fracture � Anterior interosseous nerve is mostly affected during posterolateral displacement of the distal fragment � Radial nerve is mostly affected with posteromedial displacement � Ulnar nerve is involved in flexion type of supracondylar fracture

283. Which virus among the following is least likely to cross placenta?

A. Rubella

B. Herpes simplex

C. HIV

D. HBV

Ans: D. HBV [REPEAT]

284. Feature of obstructive azoospermia is?

A. High FSH, high testosterone

B. Low FSH, high testosterone

Online support at www.pgpyrexia.co.cc

Free monthly AIIMS/ALL INDIA mock exams @ www.pgpyrexia.lefora.com.

Visit today!! Page 97

C. High FSH, low testosterone

D. Normal FSH, normal testosterone

Ans: D. NORMAL Testosterone and FSH [REPEAT]

285. Acoustic neuroma involves

A. Superior vestibular division of 8th cranial nerve

B. Auditory part of 8th cranial nerve

C. 7th cranial nerve

D. Inferior vestibular division of 8th cranial nerve

Ans: A. Superior vestibular division of 8th cranial nerve

As long as books say superior vestibular nerve as the answer, Superior vestibular division of 8th cranial nerve will remain as the answer!!!

286. Right isomerism is?

A. Asplenia

B. Two spleens

C. One spleen

D. Polysplenia

Ans: A. Asplenia

Right isomerism is also called asplenia syndrome (also called as ivemark syndrome) is characterized by duplication of right-sided structures and absence of the spleen. There are bilateral trilobed lungs containing eparterial bronchi and both atria are morphologic right atria. The liver is symmetric and transverse and the stomach is also near the midline. There can also be common atrioventricular canal, univentricular heart, transposition of the great arteries, and total anomalous pulmonary venous return. Due to the immune problems associated with an absent spleen and the complex cardiac problems the mortality rate is up to 80% in the first year.

287. A primigravida at 37 weeks of gestation with loss of engagement and Cervix 1cm dilated for the past 10hrs. What is management?

A. Sedate the patient and wait

B. LSCS

C. Amniotomy and augmentation with oxytocin

D. Induction with membrane rupture

Ans: A Sedate the patient and wait [REPEAT]

288. Following are true about carbohydrate antigen except?

A. Memory

B. Poly clonal response

Online support at www.pgpyrexia.co.cc

Free monthly AIIMS/ALL INDIA mock exams @ www.pgpyrexia.lefora.com.

Visit today!! Page 98

C. Poor immunogenicity

D. T cell independent immunity

Ans: A. Memory [REPEAT]

“This is because B cells make antibodies to carbohydrate antigens without help from T cells and so there is no consequent memory response”

289. A 70yr old presents with intemittent jerks of recent origin, EEG showing bilateral periodic spikes. What is the most probable diagnosis?

A. Hepes simplex encephalitis

B. Lewy body dementia

C. Alzheimer's

D. CJD

Ans: D. CJD

Ref: Harrison

“The constellation of dementia, myoclonus, and periodic electrical bursts in an afebrile 60-year-old patient generally indicates CJD. Clinical abnormalities in CJD are confined to the CNS. Fever, elevated sedimentation rate, leukocytosis in blood, or a pleocytosis in cerebrospinal fluid (CSF) should alert the physician to another etiology to explain the patient's CNS dysfunction”

290. A 50yr old patient presents with 2 yrs h/o recurrent abdominal pain, radiating to back, relived only by parenteral analgesic. USG & CT confirmed the diagnosis ,appropriate procedure is?

A. Vagotomy with Gastroduodenostomy

B. Vagotomy with antrectomy

C. Whipple procedure

D. Longitudinal pancreaticojejunostomy

Ans: D. Longitudinal Pacreaticojejunostomy

The Diagnosis is Chronic Pancreatitis which was medically managed.

Re: Sabiston

The two indications for surgical intervention are pain and concern about the possible presence of cancer. After the diagnosis of chronic pancreatitis has been established, surgical intervention is considered when (1) the pain is severe enough to limit the patient's lifestyle or reduce productivity, and (2) the pain persists despite complete abstinence from alcohol and administration of non-narcotic analgesics.

Puestow and Gillesby, in 1958, described an operation that involved longitudinally opening the entire duct and then invaginating the opened pancreas into a Roux-en-Y loop of jejunum. This allowed for more complete decompression but still required splenectomy. Later, Partington and Rochelle[37] modified the Puestow procedure by creating a side-to-side anastomosis between the opened duct and jejunum, thus eliminating the need for splenectomy

291. In a 5 year old child the burn area corresponding to thesize of palm is equal to

A. 1% BSA

Online support at www.pgpyrexia.co.cc

Free monthly AIIMS/ALL INDIA mock exams @ www.pgpyrexia.lefora.com.

Visit today!! Page 99

B. 5% BSA

C. 10% BSA

D. 20% BSA

Ans: A.1% BSA

Ref: Nelson

The rule of nines used in adults may be used only in children older than age 14 yr or as a very rough estimate to institute therapy before transfer to a burn center. In small burns of <10% of BSA, the rule of palm may be used, especially in outpatient settings. The area from the wrist crease to the finger crease (the palm) in the child equals 1% of the child's BSA.

292. A female presents with sings of meningitis. CSF shows gram positive bacilli. It is most probably?

A. Listeria

B. Haemophilus influenzae

C. Pneumococcus

D. Staphylococcus

Ans: A. Listeria

(Among the given options.,C and D are cocci and H.influenza is Gm(-) bacillus.

Only Listeria is the gram positive (cocco)bacilli)

293. Most useful indicator for severity of acute illness

A.Case fatalityrate

B. Prevalence

C. Incidence

D. Incubation period

Ans: A. Case fatalityrate [REPEAT]

294. A boy gets hit by a tennis ball in the eye following which he has complaints of decreased vision. Which of the following tells that blunt injury is due to the ball?

A. Optic neuritis

B. Pars planitis

C. Vitreous base detachment

D. Equatorial edema

Ans: C. Vitreous base detachment

Ref: Vitreoretinal disease: the essentials By Carl D. Regillo, Gary C. Brown, Harry W. Flynn

“The presence of Vitreous base avulsion is thought to be pathognomonic of blunt ocular trauma”

Online support at www.pgpyrexia.co.cc

Free monthly AIIMS/ALL INDIA mock exams @ www.pgpyrexia.lefora.com.

Visit today!! Page 100

295. True regarding drug resistance of MRSA?

A. Penicillinase enzyme production

B. Due to change in penicillin binding receptors

C. Plasmid mediated

D. Treated with amoxicillin clavulanic acid

Ans: B. Due to change in penicillin binding receptors [REPEAT]

296. Mr X is a chronic smoker. His family insists on quitting smoking. He is thinking about quitting, but is reluctant to do so because he is worried that on quitting he will become irritable. This is?

A. Precontemplation and preparation.

B. Contemplation and extent of sickness susceptability

C. Contemplation and cost factors

D. Precontemplation and cost factors

Ans: C. Contemplation and cost factors

Contemplation since the person is thinking about quitting and cost factors---the effects he may have to stand after quitting

In the Transtheoretical Model, change is a "process involving progress through a series of stages

� Precontemplation – "people are not intending to take action in the foreseeable future, and are most likely unaware that their behaviour is problematic"

� Contemplation – "people are beginning to recognize that their behaviour is problematic, and start to look at the pros and cons of their continued actions "

� Preparation – "people are intending to take action in the immediate future, and may begin taking small steps towards change

� Action – "people have made specific overt modifications in their life style, and positive change has occurred"

� Maintenance – "people are working to prevent relapse," a stage which can last indefinitely"

� Termination – "individuals have zero temptation and 100% self-efficacy... they are sure they will not return to their old unhealthy habit as a way of coping

Online support at www.pgpyrexia.co.cc

Free monthly AIIMS/ALL INDIA mock exams @ www.pgpyrexia.lefora.com.

Visit today!! Page 101

297. Which among the following is an early sign of magnesium toxicity?

A. Loss of deep tendon reflexes

B. Respiratory depression

C. Cardiac arrest

D. Decreased urine output

Ans: A. Loss of DTR

Read below:

� Normal plasma magnesium levels range from 1.5 to 2.5 or 3.0 mEq/L. � As plasma magnesium rises above 4 mEq/L, the deep tendon reflexes are first decreased � As plasma magnesium rises above 10 mEq/L, respiratory paralysis may occur. Heart block also may occur at

this or lower plasma levels of magnesium.

298. A 7 yr old child underwent neurosurgery for craniopharyngioma following which pituitary function was lost. Which hormone is to be replaced first?

A. Hydrocortisone

B. Thyroxine

C. Growth hormone

D. Gonadotropins

Ans: A. Hydrocortisone

Ref: http://pituitaryadenomas.com/perioperativetreatment.htm

Glucocorticoids are routinely administered to all patients following surgery for a pituitary tumour, regardless of preoperative HPA axis testing. Patients with preoperative pituitary-adrenal insufficiency usually require replacement steroids following surgery. In these patients and in those subjected to total hypophysectomy, the high dose steroids (dexamethasone 4 to 8 mg per day) are tapered to replacement levels, which are adjusted before discharge

Also,

Following surgery for a pituitary tumour, the patient who is preoperatively euthyroid may become deficient in thyroid-stimulating hormone; this is not important in the immediate postoperative period, however, because thyroxin has a plasma half-life of 7 days.

Gonadotropin deficiency does not require treatment in the immediate postoperative period.

The recombinant growth hormone is generally given subcutaneously in doses of 0.3 mg/kg per week, either daily or tri-weekly (but not in the immediate post-op period).

299. Patient prestent with high TSH, low T4, what is the diagnosis?

A. Grave's disease

B. Hashimoto's disease

C. Pituitary failure

D. Hypothalamic failure

Online support at www.pgpyrexia.co.cc

Free monthly AIIMS/ALL INDIA mock exams @ www.pgpyrexia.lefora.com.

Visit today!! Page 102

Ans: B. Hashimoto's disease

Of the given options, only opt.B fits the given scenario.

Graves’s disease has high T4

Opt, C&D have low TSH

300. Drugs used in prophylaxis of migraine are all except?

A. Propranolol

B. Flunarizine

C. Topiramate

D. Levetiracetam

Ans: D. Levetiracetam

Ref: Harrison:

“The drugs that have been approved by the FDA for the prophylactic treatment of migraine include propranolol, timolol, sodium valproate, topiramate, and methysergide. In addition, a number of other drugs appear to display prophylactic efficacy. This group includes amitriptyline, nortriptyline, flunarizine, phenelzine, gabapentin, topiramate, and cyproheptadine”

Drugs used in prophylaxis of migraine:Review

Propranolol,

Timolol,

Sodium valproate,

Topiramate,

Methysergide**

Amitriptyline,

Nortriptyline,

Flunarizine,

Phenelzine, **

Gabapentin,

Topiramate,

Cyproheptadine

** Phenelzine and methysergide are usually reserved for recalcitrant cases because of their serious potential side effects

Online support at www.pgpyrexia.co.cc

Free monthly AIIMS/ALL INDIA mock exams @

Visit today!!

A kind request to readers:

We are very happy to provide you with resource that helps you in your battle for a PG seat.If you want to appreciate our work and help us do better, kindly recall the Questions of upcoming AIIMS and PGI exams by contributing in our “EXAM RECALLER” section of pgpyrexia.co.cc

Looking forward to your participation!!

ALL THE BEST FOR THE EXAMS!!

Online support at www.pgpyrexia.co.cc

Free monthly AIIMS/ALL INDIA mock exams @ www.pgpyrexia.lefora.com

are very happy to provide you with resource that helps you in your battle for a PG seat.If you want to appreciate our work and help us do better, kindly recall the Questions of upcoming AIIMS and PGI exams by contributing in our “EXAM

www.pgpyrexia.lefora.com.

Page 103

are very happy to provide you with resource that helps you in your battle for a PG seat.If you want to appreciate our work and help us do better, kindly recall the Questions of upcoming AIIMS and PGI exams by contributing in our “EXAM